Wound and Ostomy quiz set

Réussis tes devoirs et examens dès maintenant avec Quizwiz!

A fistula between the colon and skin is known as a/an: Select one: a. Colocutaneous fistula. b. Colovesicle fistula. c. Rectovaginal fistula. d. Internal fistula.

a. Colocutaneous fistula.

What is the primary risk factor associated with Peristomal MASD? a. Obesity b. Pouching system leakage c. Large amount of wound exudate d. Urinary incontinence

b. Pouching system leakage

You have a patient with reticulated (lace-like) macules and papules that are dark maroon to purple. Which of the following terms BEST describes this type of skin lesion: a. Bruise b. Purpura c. Ecchymosis d. Varicosity

b. Purpura Correct term for extravasation (leaking) of blood into the tissues that are ≤ 1cm is purpura. Ecchymosis describes large areas of extravasation of blood, bruise is vague term, while varicosities describes veins.

Eighteen hours after the creation of an end stoma, the nurse notes that the stoma mucosa is black and flaccid. What would be the appropriate assessments and interventions? Select one: a. Check the hydration level of the patient and provide hydration, because the flaccid stoma may be the result of dehydration. b. Remove the stoma pouch and gently rub the stoma to note any surface bleeding. Notify the surgical service and prepare to examine the stoma using a small, lubricated, glass tube and flashlight. c. Encourage the patient to ambulate since this activity may precipitate bowel function. d. Remove the ostomy pouching system, cut a larger opening in the new pouching system and place a gauze pad into the pouch to prevent that plastic from contacting the stoma.

b. Remove the stoma pouch and gently rub the stoma to note any surface bleeding. Notify the surgical service and prepare to examine the stoma using a small, lubricated, glass tube and flashlight.

Nutritional interventions for wound repair, unless contraindicated, include: a. Caloric needs of 10-15 cal/kg/day b. Caloric needs of 90-100 cal/kg/day c. Caloric needs of 30-35 cal/kg/day d. Caloric needs of 20-25 cal/kg/day

c. Caloric needs of 30-35 cal/kg/day

You are providing a preoperative visit for a patient about to undergo a protecting stoma and LAR (low anterior resection). Why is a protecting stoma usually utilized in these procedures? Select one: a. The incidence of bleeding increases when an LAR is performed. b. Diarrhea may occur and a protecting stoma eliminates this problem. c. It protects the distal anastomosis by reducing the risk of dehiscence at the site. d. Performing a protecting stoma enhances perfusion to the proximal colon.

c. It protects the distal anastomosis by reducing the risk of dehiscence at the site. A protecting stoma is temporary and acts as a safety mechanism to divert effluent before an anastomosis. This protects low colonic anastomosis while healing. The incidence of anastomotic dehiscence increases the more distally the anastomosis is created.

You are observing the limbs of a new patient and note that one limb has brawny non-pitting edema from the toes to groin and positive Stemmer sign while the other is without edema and no distortion in the leg shape. What type of disorder is this patient most likely suffering from? Select one: a. Venous disease. b. Lipedema. c. Lymphedema. d. Dependent edema.

c. Lymphedema. This is a classic case of lymphedema. Lymphedema is usually unilateral from the toes to groin with brawny non-pitting edema and positive Stemmer sign (not possible to pinch fold of skin over the top of the second toe).

The most effective pouching technique for the patient with an ileostomy and a small fistula open on the skin about 5mm from the base of the stoma is to: Select one: a. Cover the fistula opening with a skin barrier paste and pouch over the fistula opening. b. Apply a skin barrier powder into the fistula track to absorb drainage and pouch over the fistula opening. c. Resize the stoma opening in the pouch to include the fistula opening. d. Place a catheter into the fistula opening, feed the catheter through the stoma opening on the pouch and anchor the catheter to the front of the pouch.

c. Resize the stoma opening in the pouch to include the fistula opening. A fistula so close to the base of the stoma has to be incorporated into the pouch opening so the pouch contains the stool as well as the fistula output.

A routine baseline assessment of the lower leg in a patient with diabetes should include: Select one: a. Segmental pressures. b. Duplex imaging. c. Sensory testing with a monofilament. d. Pulse volume recording (PVR).

c. Sensory testing with a monofilament. Sensory testing with monofilament to detect neuropathy is a critical baseline assessment.

One of the reasons scab formation and necrotic tissue are generally considered undesirable in the wound is because they: Select one: a. Keep the wound bed too moist. b. Apply more pressure to the wound surface. c. Slow the process of epithelialization. d. Stimulate excess angiogenesis and fibroblast proliferation.

c. Slow the process of epithelialization. Epithelial cells must have moisture to migrate and multiply across the wound bed from edge to edge and follicle to follicle. However, when the wound surface is dry or a scab or necrotic tissue is present, the epithelial cells are forced to burrow down to a moist layer in order to migrate across the wound bed. This will result in slower epithelialization.

What qualifies your patient with a low Braden MOBILITY subscale score for a low air loss redistribution support surface (BED)? Select one: a. Stage 4 heel ulcer. b. Stage 2 occiput ulcer. c. Stage 3 sacral pressure ulcer. d. IAD (incontinence associated dermatitis).

c. Stage 3 sacral pressure ulcer. Pressure Ulcer Guidelines outlined in your text suggest Persons with Braden mobility subscale scores of 2 or 1, existing pressure ulcers on the trunk or pelvis, and 2 available turning surfaces should be placed on a reactive/CLP support surface or an active support surface with an AP feature.

When the skin around a wound develops a whitish appearance, it can be described as: Select one: a. Desiccated. b. Excoriated. c. Indurated. d. Macerated.

d. Macerated.

A primary dressing material indicated for a wound with depth and moderate amount of exudate is a/an: Select one: a. Alginate. b. Composite. c. Hydrogel. d. Moist gauze.

a. Alginate. Alginates. Composite dressings provide some absorption but do not fill the wound bed depth. Hydrogel by itself does not fill wound depth and does not absorb exudate. Gauze can fill wound depth and absorb exudate; however, it will not absorb exudate when it is moist.

Mr. Rooke worked in a rubber plant for over 30 years. He was found to have bladder cancer involving the muscle layer and peri-vesical fat of his bladder. What type of treatment will he MOST likely have for this invasive carcinoma? Select one: a. Radical cystectomy and urinary diversion b. Intravesical treatment c. Radiation therapy d. Watchful waiting

a. Radical cystectomy and urinary diversion Invasive bladder cancer will require removal of the bladder and an ileal conduit or Indiana Pouch performed

Which of the following foods tend to help thicken loose stool? a. Rice, toast, bananas, applesauce b. Pasta, apples with skins, grapes c. Peanut butter, broccoli, beans d. Mashed potatoes, Chinese vegetables, oranges

a. Rice, toast, bananas, applesauce Remember the BRAT (Bananas, Rice, Applesauce, Toast or Tapioca) diet when you are having loose stool to help thicken it? Other foods that thicken stool include mash potatoes, pasta, cheese, peanut butter, marshmallows, crackers.

Which of the following complications is a contributing factor to the high mortality rate associated with the patient who has a fistula? Select one: a. Sepsis b. Dehydration c. Thrombophlebitis d. Pulmonary embolism

a. Sepsis

Two of the MOST important assessments to make when determining the pouching procedure and equipment to use postoperatively are: Select one: a. The size of the stoma (height, size) and the softness or firmness of the abdomen in the peristomal field. b. Whether it is a permanent or temporary type of stoma construction and odor. c. Pouching equipment most familiar to the patient and their manual dexterity. d. Patient's hand preference (right versus left) and stoma size.

a. The size of the stoma (height, size) and the softness or firmness of the abdomen in the peristomal field. The stoma height, size and where the lumen empties dictate the type of ostomy equipment we recommend. Abdominal contours influence our decisions because a soft abdomen often needs firm convexity. Patient's hand preference and odor are not the MOST IMPORTANT factors to consider in pouching system selection.

Which of the following statements about inelastic compression bandages is accurate? Select one: a. They are only effective in the ambulatory patient. b. They may be used when the ABI = 0.5 or less. c. They can only be used after edema is reduced. d. They provide sustained compression whether the patient is ambulatory or immobile.

a. They are only effective in the ambulatory patient. Inelastic compression is indicated primarily for ambulatory patients. These can be used initially in the presence of considerable edema or later with very little edema.

What is Mrs. Lang's foot deformity called when there is a rocker bottom appearance to her foot? Select one: a. Claw toes. b. Charcot's joint. c. Hammer toes. d. Onychomycosis.

b. Charcot's joint. The collapse of the arch associated with autonomic neuropathy contributes to the development of the rocker bottom foot shape known as Charcot's joint

In which of the following wounds would surgical intervention for debridement be considered urgent? Select one: a. Calciphylaxis. b. Necrotizing fasciitis. c. Pyoderma granulosum. d. Epidermolysis bullosa.

b. Necrotizing fasciitis. Immediate debridement once necrotizing fasciitis is suspected is critical to reduce the mortality of this disease and because the fasciitis spreads within a matter of minutes and hours

Type 2 skin tear is described as: a. Full thickness skin tear without an epidermal flap b. Partial thickness skin tear with viable flap with some degree of skin loss c. Partial thickness with viable epidermal flap that is completely approximated d. Partial thickness wound without an epidermal flap

b. Partial thickness skin tear with viable flap with some degree of skin loss Type 2 skin tear is partial thickness skin injury due to separation of skin layers with a viable skin flap that cannot be fully approximated due to some level of tissue loss.

As you setup a skin care program, you identify the elderly to be at risk for skin tears due to the effects of aging on: a. Tissue oxygenation b. Rete ridges c. Growth factors d. Tissue macrophages

b. Rete ridges With aging, the dermal-epidermal junction (specifically the rete ridges & dermal papillae) flatten and it is this flattening that places the elderly at risk for skin tears. Aging itself has no effect on macrophages, growth factors or tissue oxygenation that would lead to skin tear.

90-year-old female in a nursing home has a partial thickness wound with a fully approximated epidermal flap on her left forearm. Identify the type of skin damage present: a. Stage 2 pressure ulcer b. Skin tear, Type I c. Skin tear, Type II d. Skin tear, Type III

b. Skin tear, Type I Skin tear, Type I is a partial thickness wound with a skin flap that can be completely approximated at the edges, no epidermis exposed. Type II is a partial thickness skin wound where the skin flap is not completely approximated. Type III skin tear has complete loss of the epidermal skin flap with 100% exposure of epidermis/dermis.

During preoperative education, what may be the greatest concerns of an adolescent with Chronic Ulcerative Colitis (CUC) undergoing a restorative ostomy surgery? Select one: a. Knowledge regarding the surgery b. Stoma location c. Diet changes d. How to apply a pouch

b. Stoma location Adolescences are very concerned about social situations, acceptance from their peers as well as hygiene and sexuality. Clothing styles play an important part of peer and social acceptance. They may have a strong opinion on where the stoma site is located but the nurse must mark the site for optimal care of the ostomy.

Mrs. James has an eschar covered heel ulcer and you decide that the best approach to management is NOT to debride but to keep the eschar covered, dry and intact. The rationale for your decision is that the: Select one: a. wound is clinically infected. b. TcPO2 (transcutaneous oxygen) is 15 mmHg. c. peri-wound is erythematous. d. albumin level is 3.0.

b. TcPO2 (transcutaneous oxygen) is 15 mmHg. A TcPO2 level below 40 mmHg demonstrates skin/tissue hypoxia and is a sign for possible delayed healing

Which layer of soft tissue is most vulnerable to pressure over a bony prominence? a. Hypodermis b. Subcutaneous c. Dermis d. Muscle

d. Muscle

Mr. Olsen has an irregular-shaped wound that is shallow, exudative and located near the medial malleolus. This ulcer is most likely what type? Select one: a. Arterial. b. Neuropathic. c. Diabetic. d. Venous.

d. Venous. Irregular shaped and in the gaiter area (specifically near the malleolus) is "classic" for venous ulcer.

how does age alter the skins epidermal turnover or process of differentiation a) prolongs the rate of cell turnover (>28 days) b) reduces the rate of cell turnover (<28 days) c) doesnt affect the rate of cell turnover d) accelerates turn over to 5 days or less

a) prolongs the rate of cell turnover (>28 days) the process of differentiation occurs over 28 days but prolongs as we age. it can be accelerated by certain disease processes such as psoriasis

what is the primary function of intact skin? a) protective barrier b) enhance body image c) communication d) vitamin d synthesis

a) protective barrier one of the main functions of skin is the keep the outside world OUT

Intermittent claudication leg pain is characterized as pain that occurs: Select one: a. with activity and is relieved by rest. b. only during the night. c. in the absence of activity with the leg in a dependent position. d. when the leg is elevated such as when the patient is lying supine.

a. with activity and is relieved by rest. Claudication is the pain that occurs with exercise or activity and is resolved with rest.

What are the primary layers of the skin? a) hypodermis and muscle b) epidermis and dermis c) kertinocytes and fibroblasts d) macrophages and platelets

b) epidermis and dermis There are 2 primary layers. The epidermis can have 4 to 5 layers.

Which type of pouching system would be appropriate to recommend for a blind person with an ileostomy? Select one: a. 1-piece. b. 2-piece. c. Closed end pouch. d. Non-adhesive pouch.

b. 2-piece.

The WOC nurse is documenting on a 'top-down' skin injury for a patient in a long-term care facility. You explain to the staff that this type of injury refers to: a. A wound that begins at the soft tissue & bone level b. A wound that begins at the surface of the skin c. A wound that is caused by poor blood flow d. A wound that is caused by poor control of blood sugars

b. A wound that begins at the surface of the skin Top-down' skin injury is a popular term used to describe superficial skin injuries. Top-down skin injuries are due to mechanical forces such as friction, moisture or the effects of inflammation. Three common types of 'top-down' skin injuries include MASD, MARSI, and skin tears.

What primary function does vitamin C play in skin or wound repair? a. Assists in vitamin A absorption b. Capillary wall integrity c. Coagulation (clotting) d. Macrophage function

b. Capillary wall integrity Vitamin C is a water-soluble vitamin and stores can be depleted within 2-3 months. The function of vitamin C assists in iron absorption, fibroblast function (makes collagen), capillary wall integrity, immune function and is a powerful antioxidant!

You are teaching a patient with a gastrostomy tube that has a balloon. You instruct the patient to do which of the following? Select one: a. Check the balloon volume daily. b. Check the balloon volume once a week. c. Replace the gastrostomy tube every two weeks. d. Replace the gastrostomy tube once a week.

b. Check the balloon volume once a week. Checking the balloon volume weekly ensures that the correct amount of fluid is in the balloon. If the amount is less that indicated, fluid should be instilled to the correct amount and documented. Fluid can evaporate, or the balloon can develop a slow leak over time.

The presence of mucus shreds in the urine after an Indiana pouch is: Select one: a. Indicative of a urinary tract infection. b. Considered normal. c. Typical of dehydration. d. Associated with an anastomotic leak.

b. Considered normal. Mucus in the urine after any urinary diversion that uses a portion of the bowel is normal.

Of the following urinary diversions, which procedure is always considered permanent? Select one: a. Pyelostomy. b. Ileal conduit. c. Vesicostomy. d. Ureterostomy.

b. Ileal conduit. The ileal conduit is always permanent because of the intricacies in creating the conduit; a quick temporary diversion would be the ureterostomy, vesicostomy and pyelostomy.

Based on what you know about absorption and digestion within the small bowel and colon, which of the following types of stomas would you expect to produce the most corrosive effluent? Select one: a. Descending colostomy. b. Ileostomy. c. Loop transverse colostomy. d. Sigmoid colostomy.

b. Ileostomy. The most digestive enzymes are present in the jejunum portion of the bowel and then the ileal segment of the bowel. Therefore, effluent from the ileostomy will be much more corrosive than effluent from the descending or sigmoid colostomy.

Anorectal malformation in the neonate is also known as: Select one: a. Hirschsprung's disease. b. Imperforate anus. c. Turcot syndrome. d. Volvulus.

b. Imperforate anus. Anorectal malformation is a more precise name for what is also known as imperforate anus. However, imperforate anus is commonly used for this congenital anomaly.

As wound healing progresses, collagen deposition and remodeling occur and has which of the following effects? Select one: a. Reduced cytokine production. b. Increased tensile strength. c. Restored innervation. d. Increased monocyte activity.

b. Increased tensile strength. The collagen (Type 3) that was first produced has very little strength. During the remodeling phase, more resilient collagen (Type 1) is being made and the weaker collagen is broken down (hence the term remodeling). Collagen provides tensile strength to the skin so that it won't rupture under pressure.

How does MASD alter the acid mantle of the skin? a. Reduces TEWL (transepidermal water loss) b. Increases TEWL (transepidermal water loss) c. Moisture dehydrates the skin d. Moisture promotes barrier function

b. Increases TEWL (transepidermal water loss) When exposed to excessive amounts of moisture, the skin will soften, swell, and become wrinkled, all of which make the skin more susceptible to damage, this increases TEWL which in turn increases the skin's pH (change in acid mantle) thereby enhance the potential for bacterial invasion

Mr. Moore is having surgery for anorectal cancer located below the dentate line. During your pre-op visit you provide facts regarding possible effects of surgery on sexuality. This represents what level of sexual counseling in the PLISSIT Model? Select one: a. Permission b. Limited Information c. Specific Suggestions d. Intensive Therapy

b. Limited Information The WOC nurse is providing factual limited information regarding the effects of an APR (Abdominoperineal resection) upon sexuality. Permission: open ended questions that gives the patient permission to ask sexuality type questions. Specific Suggestion: WOC nurse provides specific interventions relating to sexuality i.e. empty pouch before sex or wear a pouch cover. Intensive Therapy is beyond the scope of a WOC nurse. Referral to a counselor for further sexuality concerns.

Which of the following types of stoma construction methods is NOT completely diverting? Select one: a. End stoma. b. Loop stoma. c. Separated double barrel stoma. d. Brooke ileostomy.

b. Loop stoma. The loop construction maintains the continuity of the posterior bowel wall and stool can slide across the lumen and down into the distal limb. If your patient reports small amounts of stool in mucus from the rectum, this is likely the cause.

Lymphedema is the accumulation of: Select one: a. Platelet rich fluid in the soft tissue. b. Protein rich fluid in the soft tissue. c. Cytokine rich fluid in the soft tissue. d. Inflammatory cells in the soft tissue.

b. Protein rich fluid in the soft tissue. Lymphedema is the collection of protein rich fluid in the soft tissue...which is why the edema is such a problem because the protein has an osmotic pull pulling more fluid out of the vasculature and cells.

Which statement accurately describes the characteristic and implication of necrotic tissue in a wound bed? Select one: a. Necrosis of muscle tissue typically results in the formation of stringy, yellow slough. b. Consistency refers to the cohesiveness of the debris; typically more advanced necrosis is thin and wet. c. Color varies as necrosis worsens, from white/gray nonviable tissue, to yellow slough, and finally to black eschar. d. A soft mushy leather eschar is not attached to the base and edges of wound and is sometimes mistaken as a scab.

c. Color varies as necrosis worsens, from white/gray nonviable tissue, to yellow slough, and finally to black eschar. The level and type of tissue death influence the clinical appearance of necrotic tissue. Color varies as necrosis worsens, from white/gray nonviable tissue to yellow slough, and finally to firm leathery hard black eschar.

A wound nurse is managing the leg ulcers of a patient with sickle cell disease. Which intervention is recommended as therapy? Select one: a. Paint the wound with betadine. b. Low protein diet. c. Compression wraps. d. Referral for amputation.

c. Compression wraps. Often venous insufficiency is common with sickle cell disease so the management of edema with compression is recommended

The functions of the hypodermis (subcutaneous tissue) include: a. Cushions, insulates and protects against invading infection b. Insulates, contributes to the sense of touch, and provides skin color c. Cushions, insulates and adds to the mobility of the skin over underlying structures d. Provides elasticity to the skin, protects from UV ray damage, and provides cushion

c. Cushions, insulates and adds to the mobility of the skin over underlying structures

Neonates are at risk for skin tears because neonatal skin has: a. Flexibility and suppleness b. A thickened stratum corneum with increased cohesion of epidermal/dermal cohesion c. Decreased epidermal/dermal cohesion as rete ridges (pegs) are not developed d. Decreased subcutaneous tissue and thickened stratum corneum

c. Decreased epidermal/dermal cohesion as rete ridges (pegs) are not developed Neonatal skin is at risk for skin tears because their skin is underdeveloped, they have minimal stratum corneum, decreased epidermal/dermal cohesion with the rete ridges (pegs) not developed, decreased subcutaneous tissue. Skin is not mature until 33 weeks.

Vinegar soaks to the stoma are an appropriate intervention to reduce or remove: Select one: a. Peyer's patches on the mucosa. b. Urine odor. c. Encrustations. d. Caput Medusa.

c. Encrustations.

Which of the following is NOT considered an etiologic factor for pressure ulcers? Select one: a. Shear force b. Prolonged pressure c. Friction d. Medical device-related injury

c. Friction Friction is considered more of a superficial injury to surface tissue (top down) while shear, prolonged pressure, and medical devices can cause injury of the deeper tissue (bottom up) where tissue is damaged at the muscle level and progresses outward.

A patient has developed a cluster of grouped clear vesicles on an erythematous base in the peri-rectal area which the patient describes as painful. You assess these lesions to be most consistent with: Select one: a. Candidiasis. b. Folliculitis. c. Herpes virus. d. Allergic dermatitis.

c. Herpes virus. Classic presentation of a herpetic infection.

Your patient with diverticulosis is concerned about infection and the need for surgery. However, through patient teaching you are able to correct misinformation and tell him that diverticulosis is very common and not an indication for surgery. Diverticulosis differs from diverticulitis in that diverticulitis may require surgery because it involves: Select one: a. Formation of herniations in the intestinal wall. b. High intraluminal pressures in the intestinal wall. c. Inflammation or infection of sac-like pouches. d. Low intraluminal pressures of the bowel wall.

c. Inflammation or infection of sac-like pouches. Diverticulosis is the condition that develops when diverticula form in the wall of the colon. Diverticulitis is the inflammation or infection of a diverticula.

Wearing a Medic alert bracelet is critical for which of the following patients? Select one: a. A 5-year-old with a nephrostomy tube. b. A 1-year-old with a vesicostomy. c. A 65-year-old with an ileal conduit. d. A 35-year-old with an Indiana pouch.

d. A 35-year-old with an Indiana pouch. anyone with a continent reservoir needs a medic alert bracelet to help communicate their needs to authorities in the time of an emergency

The WOC nurse may think that sexual issues are the exclusive domain of the Ostomy Care Nurse because of surgical procedures and the potential for chemotherapy or radiation. However, sexuality issues involve all scopes of WOC Nursing. What is the model that is applied when consulting with patients who may have sexuality issues? a. The Erickson Model b. Borwel's Model of Adaptation c. Roy's Model of Human Development d. The PLISSIT Model

d. The PLISSIT Model

MASD (Moisture Associated Skin Damage) affects what primary structures of the skin? a. Epidermis and or dermis b. Subcutaneous tissue c. Fibroblasts d. Macrophages

a. Epidermis and or dermis

what is the benefit in the skin maintaining an acid mantle? a) promotes skin hydration b) increases TEWL c) retards growth of skin pathogins d) promotes skin elasticity

C) retards growth of skin pathogins Bacteria likes a alkaline skin surface. Without the skin being acidic it promotes the potential growth of fungal and bacterial infections

what are the functions of the skin

Provide a protective barrier, immunological surveillance, metabolism, sensation, thermoregulation, communication for social and sexual function

Which of the following describes the loss of epidermis due to a rupture of a vesicle or bulla? a. Erosion b. Fissure c. Crust d. Excoriation

a. Erosion Erosion is the loss of the epidermis due to a rupture of a blister (vesicle or bulla). Fissure is a linear crack in the epidermis due to dry skin or fungal disorder. Crust represents a scab from dried serum or blood while excoriation is a linear abrasion from scratching.

what is TEWL

Trans epidermal water loss loss of water from the inside of the body through the epidermal layer to the surrounding atmosphere via diffusion and evaporation processes

what is the definition of MASD a) skin damage caused by prolonged exposure to various sources of moisture b) Skin damage that is due to stripping action while cleansing the skin with soap and water. c) Skin damage due to radiation dermatitis. d) Skin damage due to infiltration of medication.

a) skin damage caused by prolonged exposure to various sources of moisture Moisture-associated skin damage (MASD) is defined as inflammation and erosion of the skin caused by prolonged exposure to various sources of moisture, including urine or stool, perspiration, wound exudate, mucus, or saliva.

A bowel prep preoperatively for ostomy surgery is routinely utilized to decrease bacteria in the bowel. The section of the bowel with the greatest level of bacteria is: Select one: a. Distal portion of the colon. b. Proximal portion of the colon. c. Distal portion of the small bowel. d. Proximal portion of the small bowel.

a. Distal portion of the colon.

Ideal discharge planning helps reduce preventable re-admissions by: a. Engaging patients and family members in the transition from hospital to home b. Encouraging patients to never return c. Identifying the importance of not being readmitted d. Ordering home health care to see the patient at home

a. Engaging patients and family members in the transition from hospital to home The ideal discharge planning helps reduce preventable re-admissions by engaging patients and family members in the transition from hospital to home.

When selecting a pouching system for a patient who has an enterocutaneous fistula, the PRIMARY feature of the system that should guide your selection is: Select one: a. A sizeable surface for cutting. b. An opaque pouch film. c. Built in convexity. d. Anti-reflux valve.

a. A sizeable surface for cutting. The most important pouch feature for a fistula is a sizeable pouch opening because fistulas come in all shapes and sizes. You will usually need to cut the opening to fit the irregular shape.

You have a patient with a sigmoid colostomy who developed stomal stenosis 8 weeks postoperatively. What is the key factor that increased your patient's risk for developing this complication? Select one: a. A stomal necrosis. b. A stomal prolapse. c. A significant recent weight loss. d. Performing sit-ups.

a. A stomal necrosis. Stomal necrosis is lack of perfusion in the mucosa - this results in cell death and scarring leading to stenosis.

Gary is experiencing a recurrence of his Crohn's Disease. Which of the following symptoms is Gary most likely experiencing? Select one: a. Abdominal pain. b. Anemia. c. Bloody stools. d. Vomiting.

a. Abdominal pain. Abdominal pain is common with Crohn's; the other s/s are typical of CUC.

Which of the following surgical procedures involves a wide resection of the sigmoid colon, rectum and anus? Select one: a. Abdominoperineal Resection (APR) b. Total Proctocolectomy (TPC) c. Ileal Pouch Anal Anastomosis (IPAA) d. Low Anterior Resection (LAR)

a. Abdominoperineal Resection (APR) The abdominoperineal resection involves a wide resection of the sigmoid colon, rectum and anus in an attempt to remove surrounding tissue and lymph nodes in rectal cancer.

What are indications for surgery in your 2-year old patient with intussusception? Select one: a. Air enema reduction failure or signs of bowel necrosis. b. Vomiting and diarrhea c. Inability to eat or drink d. RUQ mass

a. Air enema reduction failure or signs of bowel necrosis.

As you examine a patient's lower extremity you observe a thin leg with dry skin and very little hair; the ABI is 0.5. From this data, what condition would you assess to be present? Select one: a. Arterial insufficiency. b. Venous hypertension. c. Neuropathy. d. Mixed arterial/venous ulcer

a. Arterial insufficiency. These are classic findings consistent with arterial insufficiency. Venous hypertension will cause leg to become edematous; the color of the leg may be discolored due to hemosiderin deposition and lipodermatosclerosis.

Mr. Nelson describes pain in his legs that occurs when he walks 3 blocks and subsides when he rests. Which of the following ulcers is associated with this type of pain? Select one: a. Arterial ulcers. b. Neuropathic ulcers. c. Pressure ulcers. d. Venous ulcers

a. Arterial ulcers. He is experiencing classic claudication symptoms which are associated with arterial insufficiency (LEAD) and arterial ulcers

The presence of dry skin and fissures in the toe web of a patient with diabetes is indicative of what type of neuropathy? Select one: a. Autonomic b. Motor. c. Sensory. d. Sympathetic.

a. Autonomic Autonomic neuropathy results in decreased sweating and results in fissures and cracks.

Diabetes causes increased risk for skin: a. Bacterial and fungal infections b. Itching and increased blood flow c. Overhydration and reduced epidermal turnover time d. Sweating and decreased folliculitis

a. Bacterial and fungal infections Diabetes leads to increased risk for bacterial skin infections, fungal infections, and itching. It causes decreased (not increased) blood flow through narrowing of small blood vessels. Sweating is decreased with neuropathy. Skin is drier due to reduced oil gland production.

A culture of safety in WOC patient care requires a health care system that supports: a. Best practice or evidence-based practice b. Adequate staffing with traditional approaches to practice c. Continuing to do things the old way, inadequate staff that "work more efficiently" d. Cost containment by restricting use of advanced technologies

a. Best practice or evidence-based practice A culture of safety in WOC patient care focuses first on supporting evidence-based practice. This, in turn, provides risk reduction.

What is the primary reason for taking the time to perform a comprehensive assessment to identify protein energy malnutrition as opposed to ordering a simple blood draw to formulate a diagnosis? Select one: a. Blood tests do not provide a 'stand-alone' assessment. b. None, blood test for malnutrition are completely reliable. c. There are no blood tests available. d. There is a high incidence of false-positive results with blood tests.

a. Blood tests do not provide a 'stand-alone' assessment. It is critical to realize that there is no laboratory test that can provide a "stand-alone" assessment of an individual's nutritional status.

You are providing some patient education for a patient at risk for a neuropathic foot ulcer in obtaining proper foot wear, which of the following is NOT an important factor when selecting proper foot wear? Select one: a. Both feet should be measured, and shoes sized to the smaller foot. b. Shoes should be sized in the afternoon to accommodate foot edema. c. Patients should stand and walk when being sized for new shoes. d. Allow for ½ inch space beyond the longest toe.

a. Both feet should be measured, and shoes sized to the smaller foot. Proper footwear may prevent callous and risk for neuropathic ulceration. Provide patients with a tracing of their feet (in a standing position) to use as a self-assessment tool for identifying shoe size, shoe design, and shoe shape appropriateness. Shoes should be sized to the larger foot!

What function does vitamin A play in maintaining skin and wound repair? a. Enhances fibroblast activity b. Formation of red blood cells c. Reduces steroid negative effects d. Energy source

a. Enhances fibroblast activity Vitamin A is a fat-soluble vitamin that is important in epithelialization, angiogenesis, cellular integrity and inflammatory response. It also can reverse the deleterious effects of corticosteroids such as prednisone. It is not a source of energy.

A 98-year-old has a pressure injury that is not healing. She has a low zinc level. How does the low zinc level affect her wound healing? a. Collagen synthesis will be delayed b. MMP levels will be low c. Increased oxygen transport d. Pre-albumin visceral stores will be depleted

a. Collagen synthesis will be delayed Zinc is an important mineral that plays a vital role in protein synthesis, enzyme systems, immune competence, and collagen formation.

A 98-year-old has a pressure ulcer that is not healing. She has a low zinc level. How does the low zinc level affect her wound healing? Select one: a. Collagen synthesis will be delayed. b. MMP levels will be low. c. Increased oxygen transport. d. Pre-albumin visceral stores will be depleted.

a. Collagen synthesis will be delayed. Zinc is an important mineral that plays a vital role in protein synthesis, enzyme systems, immune competence, and collagen formation

Body Image is defined as how one sees oneself. It may be in the view in the mirror or it may be in the person's mind. What type of body image concerns might a WOC patient experience? a. Concern about attractiveness b. Fear of opioid addiction c. Anxiety about finances d. Concern about weight gain

a. Concern about attractiveness

The superficial burn with red, unbroken skin present is best managed with: Select one: a. Cooled hydrogel dressings to the burned area. b. Hydrocolloid dressings and ice pack to the site. c. Debridement and sulfamylon cream. d. Vaseline petroleum jelly to the burn site.

a. Cooled hydrogel dressings to the burned area. Cooled hydrogel dressing will address the discomfort of the burn topically.

Which of the following groups of medications is used in the medical management of Crohn's disease? Select one: a. Corticosteroids, immune suppressants REMICADE® (infliximab) and antibiotics b. Anticholinergics, antibiotics and anti-emetics. c. Antidepressants, immune suppressants and antibiotics. d. Corticosteroids, immune suppressants and antidepressants.

a. Corticosteroids, immune suppressants REMICADE® (infliximab) and antibiotics Medical management of Crohn's disease requires anti-inflammatories (especially corticosteroids and immune suppressants) and antibiotics, typically anaerobic such a FLAGYL®(metronidazole) and ciprofloxacin if abscess is suspected to cover enteric flora. If perirectal disease is present especially perianal fistulas, REMICADE® (infliximab) will be used.

Mr. Lee was found lying at home on the floor. Upon admission, he has an intact, nonblanchable, persistent purple discoloration over the coccyx. Which describes the classification of his pressure ulcer? Select one: a. Deep tissue injury. b. Stage 1. c. Unstageable. d. Stage 3.

a. Deep tissue injury. This describes the classic appearance of a deep tissue injury (DTI). A deep tissue injury presents with a purple or maroon localized area of discolored intact skin or blood filled blister over the area where sustained pressure occurs

Which of the following statements is FALSE in effective skin preparation prior to application of an adhesive product: a. Excessive hair should be shaved from the area b. Skin should be clean and dry c. Liquid barrier film or skin sealant may be used prior to adhesive application d. Avoid the use of adhesion promoters such as tincture of benzoin

a. Excessive hair should be shaved from the area Removal of excess hair by clipping with a sterile scissors or a clipper with a disposable head is recommended in an effective skin preparation for a medical adhesive product. Shaving has been shown to increase infection risk and is associated with folliculitis. Rest of the statements are correct in skin preparation for an adhesive product.

Which statement about incontinence-related skin damage is TRUE? a. Exposure to a mixture of urine and feces is more irritating than feces alone. b. The area of skin damaged by incontinent episodes is characterized be pinpoint satellite lesions along the periphery. c. Containment garments alone are the most effective way to protect the skin from liquid stool. d. Urine with an acidic pH is more damaging to the skin than urine with an alkaline pH.

a. Exposure to a mixture of urine and feces is more irritating than feces alone. Fecal & Urinary Incontinence is more damaging to the skin than urine or stool alone. Fecal enzymes become more active when the pH of the skin is elevated from skin inflammation occurring from moisture exposure.

The primary rationale for using a skin barrier paste is to: Select one: a. Fill or "caulk" irregular surfaces at the base of the stoma. b. Increase the tack of the adhesive surface of the abdomen. c. Protect the stoma from mechanical trauma. d. Reduce the burning sensation associated with the use of skin barrier powders.

a. Fill or "caulk" irregular surfaces at the base of the stoma.

Which of the following food causes odorous urine? a. Fish, brussels sprouts, garlic b. Apples, prunes, dairy products c. Steak, celery, mushrooms d. Pork, nuts, and coffee

a. Fish, brussels sprouts, garlic Foods or beverages that cause your urine to be smell different from the usual include fish, brussels sprouts, curry, garlic, coffee, Fenugreek (herb), puffed wheat as well as poor fluid intake!

In the following examples, which one reflects demographic barriers to following a plan of care? a. Food fasting due to religious practices b. Major depression c. Poor dexterity d. Unstable gait with use of a walker

a. Food fasting due to religious practices Demographic barriers include education (ability to read/write), cultural practices (language, religious or other cultural practices such as clothing, undergarments, foods) and economics (financial resources, transportation) that may prevent patient from following the plan of care.

Which of the following situations is indicative of a wound infection? Select one: a. Friable red wound bed, increase in exudate production, periwound erythema and odor. b. The presence of a slight odor upon removal of a hydrocolloid dressing. c. Absence of periwound erythema, but purulent material present under a transparent dressing. d. A decrease in exudate production.

a. Friable red wound bed, increase in exudate production, periwound erythema and odor. Indicators of a possible wound infection include friable red wound bed, increase in exudate production, erythema, odor, pain and no sign of improvement in the wound dimensions for the past 2 weeks. A slight odor upon removal of a hydrocolloid dressing is to be expected. Because autolysis is facilitated by an occlusive dressing such as the transparent dressing, purulent-appearing fluid will develop under the transparent dressing directly over the wound bed.

Which of the following wounds will heal with scar formation? Select one: a. Full thickness wound. b. Partial thickness wound. c. Classification 2 skin tear. d. Stage II pressure ulcer.

a. Full thickness wound. Scar tissue formation only occurs when there is full thickness skin loss (remember full thickness is damage through the epidermis and dermis and into the subcutaneous or lower layers/structures). Class 2 skin tears and stage 2 pressure ulcers are partial thickness.

Which of the following is the primary purpose of a comprehensive patient assessment across the scopes? a. Gather clues regarding the potential etiology of a wound, ostomy, or continence issue b. Identify support systems to assist the patient at discharge c. Documentation of events to report to risk management d. Documentation to support referral to a dietician

a. Gather clues regarding the potential etiology of a wound, ostomy, or continence issue The primary purpose of a comprehensive patient assessment is to gather information surrounding the 'problem' you are investigating where it be wound, ostomy, or continence related. The plan of care obtained from the comprehensive patient assessment hopefully will prevent 'never events' and identify risk factors that may impact patient's ability to heal or rehabilitate successfully.

When changing a wound dressing over a full thickness wound, you notice the wound bed is shiny, moist, red and has a cobblestone appearance. What terminology would you use to document this observation? Select one: a. Granulation is present. b. Epithelialization is progressing from the wound edges. c. Hyperplasia is developing. d. A film of denatured protein covers the wound bed.

a. Granulation is present.

As you obtain your past medical history, your patient with metastatic colon cancer divulges he is having problems with keeping his skin from getting sore between his buttocks. Which of the following may contribute to his skin problem? a. History of radiation b. Use of skin care products with a pH 7.0 c. Use of a Colostomy Pouch d. Good blood sugar control

a. History of radiation Radiation, use of alkaline skin care products, and poor blood sugar control can negatively impact skin health. Wearing a properly fitting ostomy pouch to contain stool will help prevent peristomal skin dermatitis, a form of MASD

You were called to the Cancer Center to evaluate a patient receiving daily radiation to his R. neck region. You note primarily dry desquamation with small patches of weeping skin, redness and edema. Which treatment modality would NOT be appropriate for this patient? Select one: a. Hydrocolloid dressing b. Use lukewarm water and non-alkaline soap, continue bathing/showers c. Non-adherent hydrogel sheet dressing d. Apply unscented hydrophilic creams twice daily

a. Hydrocolloid dressing A hydrocolloid dressing is adhesive plus it needs to be removed prior to radiation treatment which is daily.

Common skin and tissue injury care goals include: a. Identify risk, prevent, assess, document and develop an optimal topical therapy plan b. Diagnose and treat c. Identify problem and chronically manage d. Suspect problem and monitor status

a. Identify risk, prevent, assess, document and develop an optimal topical therapy plan Common skin and tissue injuries care goals include identify risk, prevent, assess, document and develop an optimal topical therapy plan.

A 21-year-old woman is on her 5th day on the orthopedic unit following a motor vehicle accident. She suddenly develops ischemic colitis from a blood clot in the inferior mesenteric artery. Which type of stoma will she most likely have following surgery? Select one: a. Ileostomy b. Jejunostomy c. Descending Colostomy d. Sigmoid Colostomy

a. Ileostomy The inferior mesenteric artery provides the blood supply to the descending colon, the sigmoid colon and the proximal portion of the rectum.

What type of MASD is more prone to fungal or bacterial infections? a. Incontinence Associated Dermatitis (IAD) b. Peristomal Dermatitis c. Contact Dermatitis d. Periwound Dermatitis

a. Incontinence Associated Dermatitis (IAD) IAD as well as ITD are both prone to fungal and secondary bacterial infection. IAD due to organisms present on the skin and enhancement under absorptive occlusion and especially if fecal incontinence-increase in colonic bacteria/yeast. Ammonia in urine increases skin pH which supports bacterial/fungal growth. ITD- dark, moist skin folds have higher levels of micro-organisms because the skin pH is more alkaline in skin folds so supports greater bacterial growth.

Three (3) primary concerns of the ostomy patient include: Select one: a. Independence, self-care and returning to previous lifestyle. b. Work, returning to social activities and finding someone to assist with ostomy care. c. Family issues, work and independence in care. d. Sexuality, management of noise and odor and locating ostomy suppliers when traveling.

a. Independence, self-care and returning to previous lifestyle.

Of the following individuals, who is MOST at risk for developing a stomal prolapse? Select one: a. Infant with a loop transverse colostomy. b. 20-year-old male with a temporary loop ileostomy. c. 65-year-old male with an end sigmoid colostomy and significant ascites. d. 55-year-old female with a transverse double barrel colostomy.

a. Infant with a loop transverse colostomy. A loop stoma is always at highest risk of prolapsing because it is usually temporary and has limited suturing to securing the bowel to the structures within the abdominal wall and the skin. Add the increased intra-abdominal pressure of an infant crying and the risk is magnified

Moisture associated skin damage is a leading cause of peristomal skin breakdown. Which is the best description of MASD in the peristomal area? Select one: a. Inflammation and erosion of the skin adjacent to the stoma. b. Macular lesions in the presence of pruritus and pain. c. Skin erythema in the pattern of the skin barrier. d. Patches of skin loss under the adhesive of a pouching surface upon pouch removal.

a. Inflammation and erosion of the skin adjacent to the stoma.

Janet is a 19 y/o female with paraplegia due to a car accident. She has a Mitrofanoff diversion due to neurogenic bladder for 2 years. She complains of leakage from her urethra since she has been away at college. What is the MOST likely cause for the urethra urine leakage? Select one: a. Infrequent catheterization of the stoma. b. Poor pelvic muscle strength. c. Pouchitis. d. Too little fluid intake.

a. Infrequent catheterization of the stoma. The patient should be catheterizing the stoma and emptying at least every 4 hours

A patient in the outpatient wound clinic has a venous ulcer and with palpation, the skin around the ulcer feels firm and woody or hardened. This describes what condition? Select one: a. Lipodermatosclerosis. b. Lymphedema. c. Hemosiderin deposition. d. Venous dermatitis.

a. Lipodermatosclerosis. Lipodermatosclerosis is the fibrosis or hardening of the soft tissue in the lower leg and indicative of long standing venous disease

The patient at greatest risk for dehydration and electrolyte imbalance is the patient with a/an: Select one: a. Loop ileostomy and IPAA (ileal pouch anal anastomosis). b. Ileal conduit. c. Perforated bowel and loop ileostomy. d. Orthotopic neobladder.

a. Loop ileostomy and IPAA (ileal pouch anal anastomosis). The patient with a urinary diversion is NOT at greatest risk for dehydration and electrolyte imbalance. It is the patient with an ileostomy located higher in the GI tract

Moisture Associated Skin Damage (MASD) occurs across the scopes of practice for a Wound, Ostomy, and Continence care nurse. What is the primary etiology for MASD? a. Moisture and mechanical forces b. Lower skin pH and moisture c. Mechanical forces and enzymatic moisture d. Dry, flaky skin exposed to moisture

a. Moisture and mechanical forces

What is the primary etiology for MASD (Moisture Associated Skin Damage) development? a. Moisture source b. Lower skin pH c. Pressure d. Dry, flaky skin

a. Moisture source Moisture is a chemical irritant, pH rises when the skin becomes macerated which reduces the acid mantle of the skin (encourages micro-organism growth), then add friction and you have the perfect scenario for the development of MASD.

Which of the following statements about wound closure methods is TRUE? Select one: a. Musculocutaneous flaps differ from skin grafts in that they carry their own blood supply. b. Skin grafts are also known as primary closure. c. Skin grafts replace deep tissue layers such as subcutaneous tissue and muscle. d. Tissue flaps are used to provide superficial wound closure.

a. Musculocutaneous flaps differ from skin grafts in that they carry their own blood supply. The musculocutaneous flap contains its own blood supply and provides subcutaneous tissue and muscle to cover the wound surface which is particularly beneficial for the stage IV pressure ulcer

Several of these surgical urinary diversions are permanent. Which urinary diversion is usually TEMPORARY? Select one: a. Nephrostomy. b. Orthotopic neobladder. c. Ileal conduit. d. Indiana pouch.

a. Nephrostomy. Nephrostomy is typically performed to provide a temporary diversion of the urine when the ureter is obstructed and retrograde access is inadvisable or impossible.

What is the primary risk factor associated with ITD (intertriginous dermatitis)? a. Obesity b. Pouching system leakage c. Large amount of wound exudate d. Urinary incontinence

a. Obesity

Risk factors for MASD: Intertriginous (ITD) would include: a. Obesity, poor hygiene, trapped or prolonged exposure to moisture b. Mobile, excessive perspiration, poor nutrition c. Hospitalization, poor nutrition, needs assistance in care d. Resident in long-term care, diabetes, obesity

a. Obesity, poor hygiene, trapped or prolonged exposure to moisture

The comprehensive role of education in the culture of safety emphasizes an informed nursing team with WOC education that includes: a. Orientation and ongoing education for the team b. Product formularies that are posted on the care units c. Brochures and education sessions if there is time at the end of the shift d. Support from the WOC Nurse if there are questions about a WOC patient

a. Orientation and ongoing education for the team The role of education in the culture of safety for WOC nurse/care team includes: orientation and ongoing education so they are current and feel confident about how to carry out the plan of care for the WOC patient.

The hospital also benefits from engaging (educating and inviting their participation) the patient and family by: a. Overall improvements in quality and safety, improved patient outcomes b. Status quo in quality, safety and no changes in patient outcomes c. Increased payment due to increased length of stay d. Increased rate of admissions

a. Overall improvements in quality and safety, improved patient outcomes The hospital also benefits from engaging the patient and family by overall improvements in quality and safety, improved patient outcomes.

What function does IRON primarily play in skin or wound repair a. Oxygen transport b. New blood vessel formation c. Antioxidant d. Formation of white blood cells

a. Oxygen transport The function of iron is to help create new red blood cells and is part of hemoglobin. Hemoglobin is responsible for carrying oxygen to all cells. If iron is deficient, then less oxygen is available for cellular function i.e. collagen formation & phagocytic activity of white blood cells.

Mr. Carter is in ICU following coronary artery bypass graft surgery. He communicates that he is having pain. The incision on his upper right thigh is approximated and intact and covered with a transparent dressing; a slight redness is noted around the incision. Which of the following assessments place him at risk for impaired oxygen perfusion to tissues surrounding the thigh wound? Select one: a. Pain. b. Heat. c. Supplemental oxygen. d. Using a transparent dressing.

a. Pain. Pain triggers vasoconstriction and therefore reduces tissue perfusion and jeopardizes wound healing. Cold triggers vasoconstriction to further decrease blood flow. Supplemental oxygen improves oxygen perfusion. A transparent dressing does not directly change oxygen perfusion of the tissues.

Building a culture of safety for the WOC patient involves the following: a. Patient-centered communication, teamwork and education b. Instruction and procedures on infection control in the health care setting c. Utilizing antimicrobial agents or dressings in most situations to control microbes d. Restricting those with influenza or colds to visit the patient

a. Patient-centered communication, teamwork and education Patient-centered communication, teamwork and education build a culture of safety that results in decreased errors because of inconsistency between caregivers, and successful transitions from one care setting to another and increased patient/family knowledge and confidence about care.

Sexual adaptation and body image are intertwined. The WOC nurse may begin an education session that involves sexual adaptation with which of the following: a. Permission Giving: Introduce the topic and mention that most people wonder about sexual activities after surgery b. Limited Information: Basic information on sexual function. If the patient has a new stoma, information on opaque pouching systems may be provided c. Specific Suggestions: There are surgeries that could enhance sexual function. A description of surgeries would be included here d. Intensive Therapy: Therapeutic sessions that may discuss body image, sexual function, the family and beliefs could all be covered within this category of interventions

a. Permission Giving: Introduce the topic and mention that most people wonder about sexual activities after surgery Permission giving - introduce the topic and mention that most people wonder about sexual activities after surgery.

Urinary and fecal incontinence is considered a risk factor for pressure ulcer development because of what factors? Select one: a. Promotion of maceration and friction. b. Enhanced acid pH and chapping of skin. c. Less skin permeability and enhanced maceration. d. Reduced friction and increased thickening of skin.

a. Promotion of maceration and friction. Moisture of urinary incontinence increases maceration and drag of the skin thus friction is enhanced. Fecal incontinence with urinary incontinence promotes the conversion of urea to ammonia thus raising skin pH (more alkaline).

Nutritional interventions for wound repair, unless contraindicated, include: a. Protein needs of 1.25-1.5 g/kg/day b. Protein needs of 0.25-0.5 g/kg/day c. Caloric needs of 10-15 cal/kg/day d. Caloric needs of 90-100 cal/kg/day

a. Protein needs of 1.25-1.5 g/kg/day

A Hartmann's Pouch is constructed when the: Select one: a. Proximal edges of the rectal segment are over sewn and left in the pelvis following colon resection. b. Entire colon is removed, and an ileal pouch is constructed in the pelvis. c. A reservoir is constructed in the abdomen for small bowel disease. d. The cecal reservoir is constructed and effluent is emptied with a catheter.

a. Proximal edges of the rectal segment are over sewn and left in the pelvis following colon resection.

A patient has a high output enterocutaneous fistula with a pouch seal that has no leakage between changes (every 3-4 days). Removing the pouch skin barrier you notice a solid patch of raised erythematous skin that matches the size of the skin barrier. The patient is complaining of itching. What is the most likely etiology of this skin condition? Select one: a. Sensitivity or Allergy to the skin barrier. b. Mechanical irritation during removal of adhesive. c. Irritant dermatitis. d. Folliculitis.

a. Sensitivity or Allergy to the skin barrier. This description is a classic example of an allergic response: the allergic reaction will conform to the size and shape of the product to which the patient is allergic.

Which of the following treatments would be most appropriate for the management of a partial thickness wound with a fully approximated epidermal flap? a. Re-approximate edges and apply a non-adherent dressing b. Apply transparent film dressing c. Apply hydrocolloid dressing d. Utilize antibiotic ointment with gauze and Kerlix (gauze bandage wrap) roll

a. Re-approximate edges and apply a non-adherent dressing Protect, keep the wound bed moist, absorb drainage, and provide atraumatic dressing removal. The most appropriate treatment choice option, is to re-approximate the wound edges (viable epidermal skin flap) and hold the edges in place with a non-adherent dressing and secure with bandage wrap (to avoid tape on delicate skin) to absorb any seepage from the wound edge until the edges seal

Which of the following treatments would be most appropriate for the management of a partial thickness wound with a fully approximated epidermal flap? Select one: a. Re-approximate edges and apply a non-adherent dressing. b. Apply transparent film dressing. c. Apply hydrocolloid dressing. d. Utilize antibiotic ointment with gauze and Kerlix (gauze bandage wrap) roll.

a. Re-approximate edges and apply a non-adherent dressing. Treatment Goals for this wound: Protect, keep the wound bed moist, absorb drainage, and provide atraumatic dressing removal. The most appropriate treatment choice option, is to re-approximate the wound edges (viable epidermal skin flap) and hold the edges in place with a non-adherent dressing and secure with bandage wrap (to avoid tape on delicate skin) to absorb any seepage from the wound edge until the edges seal

Skin tear management includes the following topical therapy: a. Realign the flap if viable and secure with non-adherent dressing b. Debridement of the viable but dry flap and cover the area with a transparent film dressing c. Debridement of the non-viable flap and cover the area with a transparent film dressing d. Moisten the dried but viable flap and loosen the flap for debridement

a. Realign the flap if viable and secure with non-adherent dressing Viable flaps should be realigned if possible, if the flap is dry but viable, the flap should be moistened first and then realigned in place. Adherent dressings including steri-strips are not supported in use by guidelines, but are frequently utilized. Removal of the adherent dressing and/or steri-strips can cause further trauma upon removal.

Your 65-year-old patient is being taught a skin/wound care procedure. According to Erickson, the best approach to working with this age group would include strategies that: a. Recognize that other health care problems may impact their abilities b. Body image is important so be sure to provide privacy c. Select equipment that is already assembled to avoid complexity d. Include written teaching materials because the patient is likely to forget easily

a. Recognize that other health care problems may impact their abilities The patient in this age group may be fully capable of learning and remembering procedures. They need all the same types of education materials and strategies as other adults. However, it's also important to recognize that other health care problems may impact their abilities.

Your outpatient presents with pseudoverrucous lesions. His stoma size is 1" and he is wearing a 1 ¼" presized pouch. He is complaining of burning and itching. What is your next step? Select one: a. Refit the pouch size opening to cover the lesions. b. Apply alcohol to dry out the lesions on the peristomal skin. c. Check the pH of the effluent. d. Apply powder and paste to cover the lesions.

a. Refit the pouch size opening to cover the lesions.

When choosing topical therapy for the patient with massive tissue loss (e.g. burn), two priority objectives are to: Select one: a. Remove nonviable tissue and control exudate. b. Provide a moist environment and prevent shear. c. Donate moisture and eliminate dead space. d. Control odor and minimize pain.

a. Remove nonviable tissue and control exudate. Priority objective in wound management of patient with massive tissue loss is always debridement (to control bacterial load and prevent infection) and manage exudate

The PRIMARY intervention for the treatment and prevention of pseudoverrucous lesions is: Select one: a. Resize the pouching system to the size of the base of the stoma to cover up the lesions. b. Apply an antifungal powder to the lesions. c. Cauterize the lesions with silver nitrate. d. Alkalinize the urine.

a. Resize the pouching system to the size of the base of the stoma to cover up the lesions. Pseudoverrucous lesions (also referred to as hyperplasia) are the result of chronic exposure to effluent fecal or urine. To correct the situation, cover the skin by correctly sizing the ostomy pouching system

Mrs. Canfield is intubated with her HOB elevated 45 degrees. She has a Stage 3 pressure injury on her coccyx with undermining. Which of the following risk factors is most responsible for undermining in the pressure ulcer? Select one: a. Shear from having the head of bed elevated to 45 degrees. b. Friction from her turning from side to side. c. Critical colonization in the wound. d. Excess moisture in the wound bed.

a. Shear from having the head of bed elevated to 45 degrees. The undermining in a pressure ulcer is most often the result of shear forces created when the head of bed is elevated over 30 degrees.

Which of the following products can be used to protect the skin around a fistula? Select one: a. Skin barrier powder or wafer. b. Thick layers of cornstarch or Maalox. c. Transparent dressing and tape. d. Gauze dressing saturated in hydrogel.

a. Skin barrier powder or wafer. Skin protection in these patients is critical because of the corrosive characteristics of the output. Cornstarch and Maalox are not evidence-based approaches.

A patient is seen with a new skin tear and inquiries about skin tears. You explain that skin tears are the: a. Skin layers separating from friction, shear, or blunt force trauma b. Immune response from allergy to soaps c. Skin drying and forming fissures d. Blisters from burns

a. Skin layers separating from friction, shear, or blunt force trauma Skin tears are caused when the skin layers separate from friction, shear, or blunt force trauma.

Describe key assessments, differentiation and treatment of moisture associated skin damage (MASD) caused by intertriginous dermatitis (ITD), peristomal and periwound moisture, as well as incontinence associated skin damage (IAD). a. Source of moisture, location, & type of skin barrier product b. Size, depth or injury, & etiology of MASD c. Condition of skin, age, & BMI d. Type of stool, presence of infection, & condition of skin

a. Source of moisture, location, & type of skin barrier product To differentiate any type of MASD, the type of moisture and location are the primary clues for the type of MASD

An important fact to teach nurses who care for a patient with a gastrostomy tube is to: Select one: a. Stabilize the tube. b. Change the tube every 2 weeks. c. Flush the tube daily with saline. d. Flush the tube daily with water.

a. Stabilize the tube.

You are selecting a stoma site pre-op for a patient having an ileal pouch anal anastomosis (IPAA) and temporary loop ileostomy. The patient has no abdominal scars but you notice the abdomen to be soft and slightly protruding. With the patient in the supine position, you palpate for the rectus muscle, make an indelible ink mark in the RLQ, confirm they can see the site while lying down and cover the mark with a transparent dressing. What step have you omitted? Select one: a. To check the potential stoma site with the patient in a sitting and standing position. b. To ask the patient if they prefer the right side or the left side. c. To consult with the surgeon to verify the site selected would be appropriate. d. To apply a sample pouch to confirm the site is away from scars and the umbilicus.

a. To check the potential stoma site with the patient in a sitting and standing position. It is critical to assess the abdomen with the patient in the supine position, sitting and standing position. When the patient is sitting, abdominal folds will become apparent that should be avoided. When standing, any drooping of the abdomen will become apparent so that the stoma site can be marked higher on abdomen if needed to remain visible to patient.

Which of the following statement is an ideal adhesive alternative for dressing securement: a. Tubular elastic dressing b. Open-weave gauze wrap c. ACE wraps d. Montgomery straps

a. Tubular elastic dressing Tubular elastic dressings are an effective choice for dressing securement. Open-weave wrap gauze and elastic bandages such as ACE wraps are not ideal choices for dressing securement because they have the potential for slippage and create variability in sub-bandage pressure related to wrapping technique. Montgomery straps uses adhesive to adhere to skin around a wound.

Severity of Skin Damage in IAD is based on what 3 things: a. Type of irritant, duration of the exposed irritant & frequency of the exposure b. Age, type of irritant, & BMI c. Duration of exposed irritant, age, & nutritional status d. Frequency of the incontinence, steroid use, & age

a. Type of irritant, duration of the exposed irritant & frequency of the exposure Severity of Skin Damage is based on 3 things: type of irritant Duration of the exposed irritant Frequency of the exposure.

Identify the primary etiology for peristomal MASD. a. Type of stoma effluent b. Protruding stoma c. A one-piece pouching system with skin barrier d. BMI < 30

a. Type of stoma effluent

Which of the following would NOT be an intervention to reduce the risk of skin damage for patients in a neonatal intensive care unit (NICU)? Select one: a. Use an adhesive remover for loosening all tapes. b. Secure dressings with gauze wraps, or flexible tube net dressing. c. Use hydrocolloids for a base on the skin then tape to that. d. Monitor for device related pressure injuries.

a. Use an adhesive remover for loosening all tapes. Eliminate use of adhesives, solvents, sealants, tapes on neonates; limit use in pediatric; avoid antibiotic ointments. Secure dressings with gauze wraps, elastic wraps, flexible tube net dressing, and hydrocolloids for a base then tape to that

You are caring for a patient who has an ileostomy over one year (removal of their terminal ileum and large intestine). What foods groups would you promote for daily intake? a. Whole grains and foods fortified with B vitamins b. Citrus fruits and juices high in vitamin C c. Yellow or orange fruits or vegetables high in vitamin A d. Liver, lean meat, or poultry high in iron

a. Whole grains and foods fortified with B vitamins Consider for those on vegan diets or those who have small bowel resection especially the terminal ileum where B12 is absorbed.

Treatment measures include which of the following for a Type 3 skin tear? a. Steri-strips to approximate edges b. Absorbent dressing with low adhesive and high MVTR i.e. silicone foam c. Dry gauze and roller gauze to secure d. Non-adherent contact layer with gauze

b. Absorbent dressing with low adhesive and high MVTR i.e. silicone foam Treatment measures for type 3 skin tear requires the use of a non-adherent absorbent dressing with high moisture vapor transmission i.e. silicone foam and secured with non-adhesive securement device i.e. elastic tubular dressing/netting.

For the patient with a urinary or fecal diversion, a well-constructed stoma and a good location, the normal wear time for an adult would be: Select one: a. 1-2 days. b. 3-4 days. c. No schedule, change when suspected leakage. d. Depends on the patient's schedule.

b. 3-4 days.

You refer a 150 lb. (68 kg) elderly patient with a 10 lb. weight loss in the last month and stage 4 pressure injury to a registered dietitian. How much protein would most likely be recommended to help support healing of the wound? a. 50-60 grams of protein b. 80-100 grams of protein c. Tube feedings d. TPN supplementation

b. 80-100 grams of protein 150 lb. person = ~68 kgs. so, may need 82-102 gms. of protein per day! (1.25 or 1.5 grams of protein X 68 kg).

Spontaneous closure of an enterocutaneous fistula is unlikely to occur when: Select one: a. The patient's albumin level is 3.4. b. A distal bowel obstruction exists. c. The fistula output is less than 100 mL. d. The patient is on antibiotics.

b. A distal bowel obstruction exists. A bowel obstruction distal to the fistula opening will prevent spontaneous closure because the bowel contents will follow the path of least resistance. The effluent cannot evacuate through the bowel because of the obstruction

Which of the following would be at the greatest risk for developing an unavoidable pressure injury while hospitalized? Select one: a. A patient with uncontrolled diabetes and long standing peripheral neuropathy b. A patient who is at end of life and has severely impaired perfusion c. A patient with a T-10 spinal cord injury d. A patient hospitalized with advance congestive heart failure

b. A patient who is at end of life and has severely impaired perfusion The patient at greatest risk for unavoidable pressure injury is the one at the end of life with poor tissue perfusion

You and a nurse are discussing the difference between skin tears and medical adhesive related skin injury (MARSI). Upon further research you find that: a. MARSI is a type of injury that occurs ONLY with moisture accumulation under an adhesive b. A skin tear can be one type of MARSI along with tension blisters, folliculitis, epidermal stripping, etc. c. MARSI is an injury to skin which persists for 5 minutes following adhesive removal d. MARSI is caused solely by the removal process

b. A skin tear can be one type of MARSI along with tension blisters, folliculitis, epidermal stripping, etc. A skin tear can be one type of MARSI along with tension blisters, folliculitis, epidermal stripping, etc. However, skin tears can be caused by other factors than just adhesives. The removal process is one cause of MARSI along with improper application, and selection of product. MARSI is a skin injury which persists for 30 minutes or more after removal of an adhesive.

You are consulted to see a patient who has just been diagnosed with rectal cancer located at the dentate line. You will prepare teaching materials to cover content areas related to which of the following surgical procedures? Select one: a. Low anterior resection (LAR). b. Abdominoperineal resection (APR). c. Total proctocolectomy (TPC). d. Ileal Pouch Anal Anastomosis (IPAA or IAR).

b. Abdominoperineal resection (APR). The patient with rectal cancer at the dentate line has a low tumor that is almost in the anus. Removal of that tumor will damage sphincter mechanisms and result in incontinence. Therefore, a colostomy is done.

You receive a referral for a patient scheduled for an abdominoperineal resection. Prior to reviewing the chart and interviewing the patient you can anticipate his diagnosis to be: Select one: a. Transitional cell carcinoma of the bladder. b. Adenocarcinoma of the colon- rectum. c. Familial Adenomatous polyposis. (FAP) d. Diverticulitis.

b. Adenocarcinoma of the colon- rectum A patient scheduled for an abdominoperineal resection usually has rectal adenocarcinoma cancer.

When can a patient with an Indiana pouch expect to begin catheterizing the stoma for emptying? Select one: a. When the reservoir attains a 500 mL capacity. b. After a pouch-o-gram indicates the suture line is healed. c. The Indiana pouch does not require catheterization to empty. d. On the 5th day postop.

b. After a pouch-o-gram indicates the suture line is healed. About 3-4 weeks after surgery, the patient with an Indiana pouch will have a pouch-o-gram to confirm the incision lines have healed. At that point you will begin teaching self-intermittent catheterization of the stoma.

A patient is having large amounts of leakage around a Penrose drain with saturation of gauze twice a shift. The surgeon asks you to evaluate and treat the red skin. The BEST action by the WOC nurse is to: Select one: a. Irrigate the drain. b. Apply an ostomy pouch with an integrated skin barrier. c. Discontinue the drain. d. Apply a barrier ointment around the drain.

b. Apply an ostomy pouch with an integrated skin barrier. Skin exposure to effluent from a Penrose drain is common. Protection of the skin starts early with skin sealants or other forms of protectants. However, the case above describes large amounts of exudate. An ostomy pouch should be implemented to protect skin and contain drainage.

A 34-year-old female is admitted for management of pain and palliative wound care for metastatic breast cancer. During her physical exam, the wound in the left breast is a fungating mass sized at 12 cm x 10 cm x various heights with slough and scant drainage. What physiologic wound care would you recommend? Select one: a. Apply antibiotic ointment to the chest wound four times daily. b. Apply hydrogel sheets to the left breast to gently debride and decrease pain. c. Use pulsatile lavage for debridement and apply an enzymatic debrider to the left breast. d. Apply Dakin's solution to the left breast.

b. Apply hydrogel sheets to the left breast to gently debride and decrease pain. The focus of palliative wound care is to control symptoms and improve the quality of life of the patient over the goal of wound healing. Hydrogel sheets are cooling and soothing, provide autolytic debridement of slough and promote non-traumatic removal to prevent bleeding & pain

Mr. Benn's venous ulcers have now resolved completely. You have fitted him with proper below the knee compression stockings. Your discharge instructions include which of the following? Select one: a. Do not remove stockings at bedtime, only when showering or bathing. b. Apply stocking upon rising in the morning. c. Launder the stockings at least weekly. d. Do not perform ankle exercises while wearing the stockings.

b. Apply stocking upon rising in the morning. Compression stockings must be applied as soon possible in the morning to prevent ankle/leg edema that will occur shortly after the patient is in the standing position.

The recombinant PDGF (platelet derived growth factor), Regranex, is specifically indicated for which of the following wounds? Select one: a. Eschar covered pressure ulcers. b. Diabetic plantar ulcers. c. Arterial ulcers with an ABI of < 0.5. d. Chronic venous ulcers.

b. Diabetic plantar ulcers. Recombinant PDGF is a growth factor that has been approved for use with "diabetic" ulcers and specifically those that occur on the plantar surface of the foot.

Mrs. Owens arrives in your Ostomy Clinic complaining that the pouch leaks within a few hours even though she puts it on exactly how she was instructed. The stoma protrudes 2 cm and her abdomen is soft; she is slightly overweight. Which of the following steps is essential to assure proper fitting of her ostomy pouch? Select one: a. Use a skin cement to increase the adherence of the pouch. b. Assess her abdominal topography and stoma with her in the sitting position and standing position. c. Trim the skin barrier to be 1/4 inch larger than the stoma. d. Instruct her to begin using a belt.

b. Assess her abdominal topography and stoma with her in the sitting position and standing position. Abdominal assessment of the patient with a stoma requires assessment when the patient is laying, sitting, or standing to identify changes in topography. In this situation, it is likely the patient will require a firm or convexity skin barrier because the abdomen in soft.

In general, the infant is so small in weight that they have more skin surface through which to absorb chemicals placed on the skin (transepidermal absorption). Which of the following nursing interventions demonstrates an understanding of that information? Select one: a. Generous use of skin sealants to reduce the risk of epidermal stripping. b. Avoiding the use of solvents on the skin. c. Minimal use of solid wafer skin barriers. d. Using EMLA or lidocaine topically to reduce the discomfort of perineal dermatitis.

b. Avoiding the use of solvents on the skin. The solvents in skin care products such as sealants, skin barrier paste and adhesives such as skin bond cement, can be absorbed through the skin; topical analgesics can also be absorbed in greater quantities than anticipated.

Mr. Simpson has a BMI >40 and is admitted for R. side hemiplegia due to a stroke. Considering pressure redistribution needs for this bariatric patient, which of the following would be most appropriate in meeting his unique needs? Select one: a. Bariatric bed with four inch foam mattress overlay b. Bariatric bed with therapeutic mattress and microclimate control c. Standard hospital bed with pressure redistribution mattress d. Bariatric air-fluidized (high air loss) integrated bed system i.e. Clinitron bed

b. Bariatric bed with therapeutic mattress and microclimate control Mr. Simpson would benefit from a pressure redistribution mattress made for bariatrics with low friction covers and one with micro-climate control (off set increase perspiration)

Which of the following foods tend to promote loose stool? a. Applesauce, pasta, mashed potatoes b. Chinese vegetables, celery, fresh fruit with skins c. Cheese and other dairy products d. Peanut butter, toast, marshmallows

b. Chinese vegetables, celery, fresh fruit with skins Fresh & fibrous vegetables and fruits will help prevent constipation by promoting peristalsis. The other food lists are great to promote a more solid type of stool by absorbing fluid within the gut.

Mr. Green has a full thickness, extensive surgical wound that you are considering NPWT. What is a contraindication for NPWT to consider in his wound treatment plan? Select one: a. Deep, gapping surgical wound with 90% granulation and 10% necrotic tissue b. Deep, gapping surgical wound with 50% granulation and 50% necrotic tissue c. Deep, gapping surgical wound with 100% granulation tissue that bleeds moderately d. Deep, gapping surgical wound with granulation tissue but large amount of exudate

b. Deep, gapping surgical wound with 50% granulation and 50% necrotic tissue NPWT is not recommended in wounds that have > 20% necrotic tissue, untreated osteomyelitis, and wounds with malignancy in wound margins

Mr. Best has been diagnosed with lower extremity arterial disease (LEAD). What risk factor for arterial disease may have played the greatest role in atherosclerosis development? Select one: a. Alcoholism. b. Diabetes mellitus. c. Sedentary lifestyle. d. Hypothyroidism

b. Diabetes mellitus. The "big four" modifiable risk factors that play a role in the development of LEAD (lower extremity arterial disease) from atherosclerosis is hypertension, smoking, diabetes mellitus, and dyslipidemia.

Periwound MASD is potentially caused by many factors. Which of the following factors contribute to this development? a. Low levels of MMPs in chronic wound exudate b. Dressing oversaturation c. Use of absorptive dressings d. Moderate amount of serous drainage

b. Dressing oversaturation Proper use of absorptive dressings should reduce the risk of periwound maceration unless the dressing is oversaturated and sits on the skin for a period leading to maceration. Viscous drainage does not absorb as well in many dressings, contain more bacteria (toxins from them damage the skin) therefore causing more periwound damage; elevated levels MMPs or enzymes and inflammatory cytokines will digest or inflame the skin thus contribute to MASD formation.

A 32-year-old patient has several linear, hollowed-out, crusted lesions with the loss of epidermis on his left lower leg. In his visit documentation, you describe these lesions as a: a. Scale b. Excoriation c. Denuded d. Bulla

b. Excoriation Excoriation is another name for 'a scratch'. You have a loss of epidermis that is linear with a hollowed-out crusted area. This term is often used inappropriately to describe loss of epidermis with weeping of the skin. The correct term to use in that situation is denudement.

A 32-year-old patient has several linear, hollowed-out, crusted lesions with the loss of epidermis on his left lower leg. In his visit documentation, you describe these lesions as a: Select one: a. Scale. b. Excoriation. c. Denuded. d. Bulla.

b. Excoriation. Excoriation is another name for 'a scratch'. You have a loss of epidermis that is linear with a hollowed-out crusted area. This term is often used inappropriately to describe loss of epidermis with weeping of the skin. The correct term to use in that situation is denudement.

Teaching strategies for use with your WOC patient include: a. Talk to the patient and encourage your patient to search the internet for additional information b. Explain, demonstrate, reinforce, repeat, write down c. Avoid assuming patients have read and understand, correct mistakes and warn the patient about ramifications of mistakes d. Discuss non-compliance and discourage questions that may seem off-topic

b. Explain, demonstrate, reinforce, repeat, write down

The patient who has a soft abdomen and a budded ileal conduit stoma will require what characteristic in the skin barrier surface of pouching system? Select one: a. Flexibility - all flexible pouching system. b. Firm surface to provide support. c. Can be either flexible or firm pouching system. d. Need more information.

b. Firm surface to provide support. Match the abdomen to the skin barrier of the pouching surface; a soft abdomen requires the resistance of a firm pouching surface while a firm abdomen is best fitted with a flexible, soft skin barrier surface on the pouching system

What is the name of the radiographic study done when investigating the origin of an enterocutaneous fistula? Select one: a. Abdominal X-ray. b. Fistulagram. c. Barium enema. d. Colonoscopy.

b. Fistulagram.

Stomal protrusion is an important assessment in ostomy management because: Select one: a. A protruding stoma will not develop stenosis. b. Flush stomas usually require convexity because effluent is exiting the stoma at skin level. c. A protruding stoma on the abdomen can always be seen by the patient wherever it is placed. d. A protruding stoma assures adequate vascularity is present.

b. Flush stomas usually require convexity because effluent is exiting the stoma at skin level. A protruding stoma will empty directly into the pouch which is ideal because it keeps the skin barrier from soaking in either urine or stool...both of which will eventually erode and dissolve the skin barrier. Flush stomas allow effluent to exit where the skin and skin barrier meet, resulting in erosion of the skin barrier and decreased wear time.

Of the following dressings, which is best for the moderately exudative venous ulcer? Select one: a. Transparent dressing. b. Foam dressing. c. Saline soaked gauze. d. Hydrogel sheet dressing.

b. Foam dressing. The goal for this ulcer is absorption. Of these options, only the foam dressing is designed to absorb exudate.

You observe a peristomal skin breakdown where there are small erythematous areas located at the hair follicle that progress from pustules to papules and may become encrusted. What is this complication and a causative organism? Select one: a. Candidiasis - fungus. b. Folliculitis - staphylococcus aureus. c. Candidiasis - Streptococcus pyogenes. d. Encrustations - Corynebacterium.

b. Folliculitis - staphylococcus aureus. This complication is caused by bacterial invasion (staph) of the hair follicle because of shaving, friction or, occasionally, occlusion. Treatment is to decrease shaving and cleanse with antibacterial soap.

Intertriginous Dermatitis (ITD) is defined as inflammation and erosion that occurs because of: a. Dry skin areas between skin surfaces (i.e. fingers or toes) b. Friction & moisture between skin folds c. Chemical injury to the skin d. Occlusion used with skin barriers

b. Friction & moisture between skin folds

Which of the following statements concerning the management of a mixed venous/arterial ulcer with an ABI of .75 is TRUE? Select one: a. Graduated compression is contraindicated. b. Graduated compression can be provided at a modified level of support. c. Graduated compression should be used temporarily to reduce edema and then discontinued. d. Graduated compression should be provided only with compression stockings.

b. Graduated compression can be provided at a modified level of support. The level of compression when the ABI is >0.5 to <0.8 is modified in the 23-30 mmHg range such as with short stretch wraps.

Which of the following statements is true for the Kock continent ileostomy? Select one: a. It is the preferred surgical procedure for Crohn's Disease. b. Nipple valve of ileum proximal to the stoma is the continence mechanism. c. The preferred stoma site is in the left lower quadrant of the abdomen. d. It is a surgical procedure that is done more often than the Ileoanal Reservoir (IAR/IPAA).

b. Nipple valve of ileum proximal to the stoma is the continence mechanism. The Kock continent ileostomy is continent because of the intussusception of the distal ileum (nipple valve) into the ileal pouch (Kock pouch).

The patient with a new skin tear wonders what caused her to be at risk for a skin tear. Your response is: a. Her active lifestyle, agility, and good nutritional status b. Her elderly age, use of steroids for chronic obstructive pulmonary disease, and recent weight loss c. Recent falls, good hydration, and use of moisturizers d. Immobility, use of pH balanced cleansers and use of moisturizers

b. Her elderly age, use of steroids for chronic obstructive pulmonary disease, and recent weight loss Common risk factors for skin tears include age (elderly), female, Caucasian, polypharmacy, corticosteroid use, chronic/critical disease, inadequate nutrition, previous skin tears. Mobility risk factors for skin tears include stiffness and spasticity, history of falls, immobility, dependence for ADLs, and use of assistive devices.

What factors most likely play the greatest risk in the development of peristomal MASD? a. Low pH, solid stool & pouch adherence b. High pH, liquid stool & eroded pouch barrier c. Acid urine & pouch adherence d. Alkaline urine & eroded pouch barrier

b. High pH, liquid stool & eroded pouch barrier Liquid stool with a high pH is more corrosive than alkaline urine alone. Digestive enzymes are present. An eroded pouch skin barrier creates the 'perfect storm' for skin to erode quickly with urine but more so with high alkaline stool.

An elderly female developed a dehisced wound following abdominal surgery for a bowel obstruction. Which of the following conditions BEST explains what placed her at risk for developing a wound dehiscence? Select one: a. Blood glucose levels ranging from 90 to 150. b. History of radiation in the surgical field. c. A prealbumin level of 28. d. No postop dressing used.

b. History of radiation in the surgical field. Irradiated tissue's microvascular circulation is negatively impacted as the year's pass and will place a person at risk for dehiscence when included in the operative field

Which of the following dressings would be most appropriate for a patient with a full-thickness pressure ulcer with significant undermining and a dry wound bed? Select one: a. Wet-to-dry gauze. b. Hydrogel impregnated gauze. c. Hydrocolloid wafer. d. Alginate.

b. Hydrogel impregnated gauze. The goal for this ulcer is to donate moisture, fill the dead space, and the dressing be retrievable. the hydrogel impregnated gauze is appropriate because it will fill the dead space, be retrievable and provide moisture.

Understanding the physiology of skin when it is injured by MASD is an important concept. Skin that is occluded by an absorptive type product (i.e. adult brief) results in what altered skin barrier effect? a. Decrease in TEWL (trans-epidermal water loss) & skin pH b. Increase in TEWL (trans-epidermal water loss) & skin pH c. Less perspiration d. Reduced friction

b. Increase in TEWL (trans-epidermal water loss) & skin pH Greater potential occurs for perspiration if the skin is occluded and evaporation of heat generated moisture is not allowed. TEWL is elevated with skin that is overhydrated as well as skin pH. Note: Increase drag or friction can occur with moist skin. Think of the analogy of sliding down a slide with a wet swim suit

Mr. Brown has mild to moderate urinary incontinence and limited mobility. Which of the following will most likely occur due to the effects of prolonged moisture on his peri-genital skin? Select one: a. Thickened and strengthened collagen bonds. b. Increased risk of mechanical injury. c. Allergic reaction to incontinence pad. d. Stabilization of skin normal flora.

b. Increased risk of mechanical injury. Urine in itself is not caustic but can macerate the skin and with the combination of the unchanged undergarment enhances mechanical injury (friction) and IAD.

In which of the following situations would you normally expect to see intermittent, brief episodes where the stoma becomes dusky, then "pinks up" again? Select one: a. 25-year-old with a loop temporary ileostomy who is dehydrated. b. Infant with a transverse colostomy during crying episodes. c. 50-year-old with a sigmoid colostomy and has had history of laxative abuse. d. 55-year-old woman with an ileostomy and recurrence of Crohn's disease.

b. Infant with a transverse colostomy during crying episodes. It should be expected that when a baby cries, they will shunt blood temporarily away from the bowel ever so briefly.

The surgical treatment of choice for the patient with a carcinoma in the upper and middle third of the rectum is: Select one: a. APR (abdominoperineal resection) b. LAR (low anterior resection) c. Proctocolectomy with ileostomy d. IPAA (ileal pouch anal anastomosis)

b. LAR (low anterior resection)

When designing educational materials for the patient, formatting guidance to follow would include: a. Use black and white print as the patient's eyes may not adjust to the reading material b. Limit the amount of material utilized (more is not better), font size should be 12-14 c. Provide comprehensive materials that the patient may read on his own after discharge d. Provide phone numbers for emergency contact after discharge, include anatomy and physiology as well as procedures in the content

b. Limit the amount of material utilized (more is not better), font size should be 12-14

The preoperative preparation for the surgical repair of an extensive wound should include which of the following interventions to reduce or control cofactors? Select one: a. Shave the preoperative site at home prior to hospital admission. b. Maintain blood glucose of 130 mg/dl or less (or tight glycemic control). c. Daily prealbumin levels to monitor nutritional status. d. Avoid the use of prophylactic antibiotic use prior to surgery.

b. Maintain blood glucose of 130 mg/dl or less (or tight glycemic control). Wound healing is optimized when blood glucose levels are 130 or less. This value may be lower based upon the patient and primary care provider. Tight glycemic control is the goal

An ambulatory patient with insulin dependent diabetes has an ulcer located on the plantar surface of the left foot over the third metatarsal head. The ulcer is 1 cm in diameter, with a dry red surface. The patient denies any pain in the ulcer. What type of wound does this most likely represent? Select one: a. Venous Ulcer. b. Neuropathic Ulcer. c. Shear Ulcer. d. Arterial Insufficiency Ulcer.

b. Neuropathic Ulcer. Diabetic neuropathic ulcer is diagnosed primarily because of the location (on the plantar surface of the foot), diabetes (increased risk of neuropathy) and the absence of pain (sensation)...certainly not a location for shear, venous or arterial.

You have a 3-year-old female toddler with spinal bifida who will be having a permanent colostomy due to neurogenic bowel. Which of the following would be most appropriate to facilitate adaptation to the new ostomy preoperatively? Select one: a. Have her apply the pouch to her tummy b. Offer her a pouch to apply to her doll c. Have her practice cutting the opening to her pouch d. Demonstrate how to empty the pouch

b. Offer her a pouch to apply to her doll This method demonstrates that the WOC nurse understands that being in the hospital and having surgery can be very frightening to a child thus using a pouch to stick onto the patient's favorite doll's abdomen is age appropriate. Patient is receiving information that she will need to have a pouch on her tummy and recognizes her feelings of fearfulness. Parents will be the ones applying the pouch, but the child can help organize supplies, remove adhesives, or with whatever they are comfortable in doing postoperatively.

Which of the following statement is true regarding MARSI (medical adhesive related skin injury) damage? a. Full thickness skin injury due to mechanical force of pressure b. Partial thickness skin injury due to friction or moisture c. MARSIs are not usually preventable d. Shave all hair prior to the application of an adhesive product

b. Partial thickness skin injury due to friction or moisture MARSI (medical adhesive related skin injuries) are partial thickness skin injuries that occur with friction or moisture. Proper adhesive application and removal and reduce MARSIs. Hair should be clipped, not shaved prior to application of adhesive product to reduce folliculitis.

You have a patient in acute care who has multiple risk factors for skin breakdown. Which of the following statements are true regarding frequency of skin assessments? Select one: a. A head to toe skin assessments should be done only upon initial admission. b. Patients at risk for skin break down should have at least a daily skin inspection. c. Skin inspection should be performed twice an eight-hour shift for those at risk. d. Skin inspections should only be done daily if the patient has a wound.

b. Patients at risk for skin break down should have at least a daily skin inspection. At least a daily skin inspection is recommended for patients at risk for skin integrity issues. Frequency of reassessment is dependent upon many variables i.e. care setting, co-morbidities, immunosuppression, tissue tolerance, continence status, nutrition, mobility etc. thus frequency of assessment may change over time.

You have a patient in acute care who has multiple risk factors for skin breakdown. Which of the following statements are true regarding frequency of skin assessments? a. A head to toe skin assessment should be done ONLY upon initial admission. b. Patients at risk for skin breakdown should have at least a daily skin inspection. c. Skin inspection should be performed twice an eight-hour shift for those at risk. d. Skin inspections should only be done daily if the patient has a wound.

b. Patients at risk for skin breakdown should have at least a daily skin inspection. At least a daily skin inspection is recommended for patients at risk for skin integrity issues. Frequency of reassessment is dependent upon many variables i.e. care setting, co-morbidities, immunosuppression, tissue tolerance, continence status, nutrition, mobility etc. thus frequency of assessment may change over time.

Select the statement that is most true regarding IAD (incontinence associated dermatitis) management. a. Skin barrier pastes should be removed entirely following a fecal incontinent episode. b. Petroleum based products may reduce the absorbency of disposable undergarments. c. Humectants are an important ingredient in incontinence skin care products. d. Soap and water should be used for incontinence skin care management.

b. Petroleum based products may reduce the absorbency of disposable undergarments. Petroleum based products do reduce the absorbency of disposable undergarments (adult briefs) by clogging the absorptive properties of the product and reducing the breathability of the product. Skin barrier pastes should not be removed entirely at the time of soiling because in doing so may cause more friction damage to the skin. Humectants add more moisture to the skin so is contraindicated in skin that is already over hydrated. Soap and water tend to be more stripping and many soaps have a high pH thus reduces the acid mantle of the skin and increases the risk for organism growth

Ms. Anderson has a diagnosis of Caput Medusae in the WOC Nursing chart. You would expect to see protuberant veins in the peristomal field which are caused by: Select one: a. Chronic exposure to effluent. b. Portal hypertension. c. Stomal prolapse. d. Parastomal hernia.

b. Portal hypertension.

Your outpatient is seen for follow-up and complaining of pouch leakage and extreme pain in the peristomal area. Upon exam, you see a reddish-purple (violaceous) discoloration with painful open lesions around the stoma. What are you looking at? Select one: a. Pseudoverrucous lesions b. Pyoderma gangrenosum (PG) c. Mucosal transplantation d. Encrustations

b. Pyoderma gangrenosum (PG)

The typical site to mark for the Indiana Pouch is: Select one: a. RUQ. b. RLQ. c. Umbilicus. d. LLQ.

b. RLQ.

The care plan for the patient with a pressure risk assessment score (Braden Scale) of 9 and on a therapeutic support surface should always include: Select one: a. Maintaining HOB at 45 degrees or less. b. Reposition the patient every 2-4 hours depending upon tissue tolerance. c. Use an artificial sheepskin under the coccyx to prevent shear. d. Avoid the use of absorbent underpads in patients suffering incontinence.

b. Reposition the patient every 2-4 hours depending upon tissue tolerance. Regardless of the support surface, best practice recommends repositioning every 2 hours as the standard of care

Successful adaptation is defined by Black as being the ability to: a. Provide self-care b. Resume everyday activities and social relationships c. Live independently and provide self-care d. Learn new hobbies and participate in new sports

b. Resume everyday activities and social relationships

As you setup a skin care program, you identify the elderly to be at risk for skin tears due to the effects of aging on: Select one: a. Tissue oxygenation. b. Rete ridges. c. Growth factors. d. Tissue macrophages.

b. Rete ridges. With aging, the dermal-epidermal junction (specifically the rete ridges and dermal papillae) flatten and it is this flattening of the rete ridges that places the elderly at risk for skin tears. Aging itself has no effect on macrophages, growth factors or tissue oxygenation that would lead to skin tear.

You are called to the neonatal intensive care unit regarding appropriate products to use on the skin of neonates. Which of the following products would be considered safe to use on the neonate's skin? a. Triple antibiotic ointment b. Silicone c. Adhesive removers d. Balsam of Peru

b. Silicone Silicone is the only product deemed safe on neonatal skin. The rest of the products are common ingredients that may cause contact dermatitis or allergic irritant dermatitis not only in the neonate but in adult skin as well.

You are called to the neonatal intensive care unit regarding appropriate products to use on the skin of neonates. Which of the following products would be considered safe to use on the neonates skin? Select one: a. Triple antibiotic ointment. b. Silicone. c. Adhesive removers. d. Balsam of Peru.

b. Silicone. Silicone is the only product deemed safe on neonatal skin. The rest of the products are common ingredients that may cause contact dermatitis or allergic irritant dermatitis not only in the neonate but in adult skin as well.

An etiology for peristomal MASD would be: a. Friction and moisture from skin folds b. Skin exposure to effluent (urine or stool) c. Mechanical injury in the peristomal area from forcible adhesive removal d. Output that is 6.5 pH

b. Skin exposure to effluent (urine or stool)

90-year-old female in a nursing home has a partial thickness wound with a fully approximated epidermal flap on her left forearm. Identify the type of skin damage present: Select one: a. Stage 2 pressure ulcer. b. Skin tear, Type I. c. Skin tear, Type II. d. Skin tear, Type III.

b. Skin tear, Type I. Skin tear, Type I is a partial thickness wound with a skin flap that can be completely approximated at the edges, no epidermis exposed. Type II is a partial thickness skin wound where the skin flap is not completely approximated. Type III skin tear has complete loss of the epidermal skin flap with 100% exposure of epidermis/dermis.

A patient with paraplegia is in the wound clinic to have a new seating surface re-evaluated. You notice a serous fluid filled blister on the left heel which you document as a pressure ulcer in what Stage? Select one: a. Stage 1. b. Stage 2. c. Stage 3. d. Stage 4.

b. Stage 2. A serous fluid blister is always a Stage 2. In contrast, a blood filled blister is a sign of deep tissue injury (DTI) which is unstageable until the depth of tissue destruction can be fully identified

Mr. Moore had an APR (abdominoperineal resection) for colorectal cancer 3 days ago. He is on a low-residue diet and taking pain medication for postoperative surgical pain. The WOC nurse notes that his stool is formed. What would be MOST appropriate to recommend for this patient? Select one: a. Dried fruit i.e. Prunes b. Stool softeners i.e. Colace c. Bulking agents i.e. Metamucil d. Stimulants i.e. Senna

b. Stool softeners i.e. Colace Mr. Moore is on a low-residue diet (post-op day #3) and pain medication, so the most appropriate choice is to use a stool softening agent like Colace

The wound care nurse is investigating why modalities and treatments used in the management of wounds are less effective with spinal cord-injured (SCI) patients. What is one reason that has surfaced? Select one: a. There is an increase in sympathetic innervation causing enhanced perfusion of the skin and soft tissues. b. TcPO2 levels may be up to five times lower below the level of injury than above, decreasing oxygen to the wound. c. Collagen is not excreted as quickly through the urine of a person with SCI causing a buildup in the wound bed. d. SCI patients have a significantly higher concentration of fibronectin, which counter acts the function of collagen.

b. TcPO2 levels may be up to five times lower below the level of injury than above, decreasing oxygen to the wound. TcPO2 levels may be up to five times lower below the level of injury than above, decreasing oxygen to the wound.

Which of the following statements about bladder cancer is TRUE? Select one: a. A radical cystectomy and ileal conduit is the treatment of choice for most bladder cancers. b. The most common type of bladder cancer is transitional cell. c. The earliest symptom of bladder cancer is difficulty in urinating. d. Sexual dysfunction is a rare complication following removal of the bladder.

b. The most common type of bladder cancer is transitional cell The most common cell type for bladder cancer is transitional cell. Anytime the bladder is removed especially for cancer, a wide resection is required to remove surrounding nodes and tissue that may be involved; consequently damage to the pudendal nerve occurs

During the first routine pouch change after an ileal conduit has been created, which of the following observations is considered normal? Select one: a. The stoma is dusky at the mucocutaneous junction. b. The stoma is edematous and pink. c. The stoma has no output. d. A constant trickle of blood from the mucocutaneous junction is present.

b. The stoma is edematous and pink. All stomas will be edematous and pinkish red immediately after surgery; may also be described as pale red simply because of all the edema.

The nurse is changing a pouching system and doing a teaching session for a new patient on the 4 postoperative day. Upon pouch removal, a separation of the mucocutaneous juncture is observed. What would you tell the nurse about why this complication may occur? Select one: a. The sutures were not completely circumferential at the stoma base. b. This most likely occurs because this patient has been on corticosteroids preoperatively. c. The presence of liquid effluent in the pouch may disrupt the sutures. d. Removal of the pouching system causes a mechanical force that can dislodge sutures.

b. This most likely occurs because this patient has been on corticosteroids preoperatively. Systemic causes are the most common reasons for this complication of the healing process. (Sepsis, corticosteroids, diabetes, malnutrition, infection, stoma necrosis, and recurrence of disease).

Which of the following causes of massive tissue loss is most commonly associated with a severe drug reaction? Select one: a. Epidermolysis bullosa (EB). b. Toxic Epidermal Necrolysis (TEN). c. Graft versus Host Disease (GVHD). d. Staphylococcal Scalded Skin Syndrome (SSSS).

b. Toxic Epidermal Necrolysis (TEN). TEN is most commonly precipitated by drug reaction such as Dilantin

Mr. Smith has significant edema to his lower extremities. What is the preferred method to measure his edematous legs to see if it is reduced over time? a. Measure calf and ankle circumference same time of day b. Use a "point of reference" so the circumference is measured the same way each time c. Estimate the amount of edema by pressing your finger gently into the pre-tibial region d. Be sure the same person measures the circumference each episode

b. Use a "point of reference" so the circumference is measured the same way each time Use a 'point of reference' to ensure that the calf and ankle measurements are measured consistently at the same level. This allows a more accurate way to assess edema amount no matter who is assessing.

The patient who is NPO for several days is at risk for atrophy of which of the following structures in the mucosal surface of the small bowel? Select one: a. Ligament of Treitz. b. Villi. c. Ampulla of Vater. d. Sphincter of Oddi.

b. Villi. Villi are small, finger-like projections that protrude from the epithelial lining of the small bowel wall to expand the absorptive surface of the small bowel.

Mrs. Anderson has dependent rubor and elevation pallor of her right lower extremity. The systolic pressure in her R. arm is 170 mmHg; L arm is 165 mmHg, R. dorsal pedis systolic pressure was 100 mmHg and the posterior tibial pressure was 95 mmHg. What is Mrs. Anderson's ABI for the RLE? Select one: a. .056 b. 1.7 c. 0.59 d. 1.65

c. 0.59

Which of the following indicates the patient with a wound needs further nutritional assessment? An unintended weight loss of: a. 5 lbs. in 30 days b. 5% in 60 days c. 10% in 6 months d. 10 lbs. in 6 months

c. 10% in 6 months Protein calorie malnutrition and inadequate nutrient intake HINDERS wound repair. Unintended weight loss of ≥ 5% in 30 days or 10% in 6 months indicates a nutritional deficit.

You have a patient with a healing venous ulcer that has become highly exudative and you suspect critical colonization. An appropriate option for topical care would be: Select one: a. A polyurethane foam dressing. b. A hydrogel impregnated gauze dressing. c. A dressing that absorbs exudate and is antimicrobial. d. A gauze dressing dampened with 1/4th strength Dakin's (hypochlorite) solution.

c. A dressing that absorbs exudate and is antimicrobial. The goal for this ulcer is absorption and when critical-colonization is suspected, an antimicrobial product i.e. silver or cadexamor iodine should be used to reduce the bacterial burden in the wound. Dakin's of this strength is cytotoxic (will damage cellular structure of healing cells) but could be used for a limited period of time when critical colonization is suspected; but does not allow for absorption of exudate.

Immediately after surgery, the patient with an orthotopic neobladder can expect to have: Select one: a. Every two-hour catheterizations of the reservoir to prevent mucus plug. b. A "band-aid" type of dressing placed over the abdominal stoma. c. A urethral catheter and a suprapubic catheter. d. Normal sensations of the need to empty the bladder.

c. A urethral catheter and a suprapubic catheter. All continent diversions will have catheters for urine drainage immediately postop to protect anastomosis. The Neobladder will have a suprapubic catheter (24 Fr. Malecot) to drain and irrigate the reservoir (Neobladder) to keep mucous manageable. A urethral catheter (22 Fr. Foley) is placed to drain and irrigate the reservoir until all suture lines between the neobladder and urethra are well healed. It is kept in place at least 2-3 weeks post-surgery. The Malecot catheter is removed once the patient is proficient at emptying their neobladder per the urethra.

Which of the following assessments is most indicative of an arterial ulcer? Select one: a. The presence of pain. b. Absence of edema in the leg. c. An ABI of 0.7. d. Absence of a ruddy, red wound bed.

c. An ABI of 0.7. An ABI of less than 0.9 is indication that the wound has an arterial insufficiency or LEAD (lower extremity arterial disease)

5 cm and is shallow. As is common with PG uclers, it has high exudate levels, is extremely painful with violaceous (purplish) borders. The wound base has slough mixed in with islands of granulation tissue. What recommendation can be made for topical therapy? Select one: a. Apply a skin substitute such as Apligraft. b. Culture the wound and then apply silver sulfadiazine (SSD) twice daily secured with rolled gauze. c. Apply a hydrofiber and cover with nonadhesive foam dressing with gentle wrap. d. Apply a hydrocolloid dressing and change twice a week.

c. Apply a hydrofiber and cover with nonadhesive foam dressing with gentle wrap. absorption, autolytic debridement & reduce pain. The hydrofiber will absorb the exudate, gelling of the hydrofiber will provide painless removal and autolytic debridement and when covered with the foam will increase absorption.

You are performing a wound assessment on a patient with a plantar foot ulceration. Wound assessment reveals a full thickness ulcer, with scant amount of serosanguinous drainage. You note that the wound base is pale pink and there is a tunnel at 3 o'clock. The wound measurements are unchanged in the last 2 week's wound assessments. The patient denies pain with palpation but you note mild periwound erythema. What would be your BEST response in the developing the plan of care for this patient? Select one: a. Apply an antimicrobial hydrofiber rope into the tunnel. b. Apply topical antibiotic ointment with a gauze strip wicked into the tunnel. c. Apply a silver hydrogel impregnated gauze strip wicked into the tunnel. d. Call primary care provider for oral antibiotics.

c. Apply a silver hydrogel impregnated gauze strip wicked into the tunnel. Goal for this ulcer is to maintain the moist wound environment and add a topical antimicrobial agent. No progress in this wound in the last 2 weeks plus mild periwound erythema may be a sign of critical colonization; minimal drainage thus a silver hydrogel to donate moisture to wound with a wick to accommodate the tunnel is the BEST topical therapy as the primary dressing.

Of the following, which would be most appropriate as a primary dressing for an exudative pressure ulcer with a depth of 4.5 cm? Select one: a. Amorphous hydrogel. b. Hydrocolloid wafer. c. Calcium alginate. d. Composite dressing.

c. Calcium alginate. The goal for this ulcer is to fill the dead space and absorb the drainage. Amorphous hydrogel delivers moisture to the wound bed and does not absorb drainage. Hydrocolloids are for light exudative & shallow wounds. Calcium alginate offers drainage absorption and fills the depth of the wound. Composite dressing may be used as a secondary dressing over primary dressing but not appropriate as the primary dressing in exudative wounds with depth.

Preoperative preparation for a patient with paraplegia who is scheduled for a myocutaneous flap closure of a Stage 4 sacral pressure ulcer includes which of the following interventions? Select one: a. Hyperbaric oxygenation to enhance perfusion of the surgical site. b. Intravenous corticosteroids to reduce inflammation at the surgical site. c. Control of spasms to prevent tension on the incision. d. Topical antibiotic ointment to reduce bioburden in the wound.

c. Control of spasms to prevent tension on the incision. Preoperative preparation for the patient undergoing a myocutaneous flap requires the control of muscle spasms...a common problem in the patient with spinal cord injury and paralysis.

An inpatient is referred to your Wound Care Service who has an ABI of 0.5 and an eschar covered ulcer on both heels; the eschar is hard and non-fluctuant. After reviewing the chart and examining the patient, you recommend pressure relief for the heels, but do not recommend debridement. What is the rationale for your decision? Select one: a. The eschar provides a moist environment that allows epithelialization to occur. b. Debridement is not urgent and can be done in an outpatient setting once he is discharged. c. Debriding dry, stable, non-infected eschar in a patient with impaired arterial perfusion will increase risk of infection. d. The patient will need intravenous antibiotics for at least 2 days before debridement can be done safely.

c. Debriding dry, stable, non-infected eschar in a patient with impaired arterial perfusion will increase risk of infection. Once the eschar is removed, the risk of a soft tissue infection is greatly increased in the ulcer with arterial insufficiency. The ulcer on a heel that is eschar covered and is not fluctuant or indicative of an infection should be left intact as it provides a protective covering and painted with betadine to reduce the bacterial load.

Negative pressure wound therapy is an appropriate option for which of the following wounds? Select one: a. Stage 4 pressure ulcer with possible osteomyelitis. b. Coccyx pressure ulcer with 20% granulation tissue and 80% eschar. c. Dehisced abdominal incision with copious exudate and mild periwound erythema. d. Dehisced abdominal wound draining fecal material and a suspected enterocutaneous fistula.

c. Dehisced abdominal incision with copious exudate and mild periwound erythema. NPWT provides excellent drainage control and contraction of a mostly clean wound.

Core concepts of patient and family centered care involve: a. Privacy, dignity and respect in care b. Alone time for the family and patient time to process information c. Dignity and respect, information sharing and collaboration d. Sympathy, information sharing and telling them what to do next

c. Dignity and respect, information sharing and collaboration Core concepts of patient and family centered care involve dignity and respect, information sharing and collaboration.

Mr. Simpson in the above case scenario is incontinent at least once a shift. You note that he has significant erythema in his peri-genital region with the presence of a macular-papular rash as well as discreet pin point lesions in peripheral skin area. Which of the following would be the best incontinence care treatment? Select one: a. Incontinence cleanser with a dimethicone barrier after each incontinent episode b. Disposable cleansing cloth with a zinc oxide/dimethicone barrier after each incontinent episode c. Disposable cleansing cloth and an application of zinc oxide barrier with an antifungal d. Incontinence cleanser after each incontinence episode & spray area with a skin sealant every 3 days

c. Disposable cleansing cloth and an application of zinc oxide barrier with an antifungal Mr. Simpson skin description is classic for mild/moderate IAD that is complicated with candidiasis

why is the brick and mortar structure key?

it is important for maintaining skin moisture and preventing TEWL

Detubularization of the bowel is a surgical technique used in continent diversions that involves surgically opening a piece of bowel length-wise to disrupt peristalsis. This procedure is utilized on some continence valves also. The rationale for detubularization in the continent diversion reservoir (pouch) is to: Select one: a. Increase the pressure to create a low volume reservoir. b. Decrease the risk for mucus production. c. Disrupt muscular contractions to reduce reservoir pressure. d. Decrease the potential for UTI.

c. Disrupt muscular contractions to reduce reservoir pressure. Detubularization allows the reservoir (pouch) to fill and expand at low pressure.

According to Erickson, when teaching a preschooler - Ages 3-5 - the BEST approach to working with this age group would include the use of: a. Parent-focused teaching b. Dolls and play c. Dolls, play and assist in care d. Large print materials with primary colors

c. Dolls, play and assist in care Dolls, plan and assist in cares is the correct answer. This may involve putting equipment together or unpacking supplies, cleaning skin, or some activity in cares where the child can be successful. They usually can't read yet.

What diagnostic test is considered the "Gold Standard" for diagnosing LEVD (lower extremity venous disease)? Select one: a. ABI (ankle brachial index). b. TcPo2 (Transcutaneous oxygen pressure measurement). c. Duplex ultrasound. d. Segmental limb pressure.

c. Duplex ultrasound. Duplex imaging is the gold standard for assessing venous disease and produces an ultrasound like image of the blood flow through a vein so that reflux, obstruction or abnormal vein walls can be identified.

Upon your assessment of Mrs. Jones sacral pressure ulcer, you note a lack of reduction of size in a clean granulating wound for the last 4 weeks thus suggested the use of electrical stimulation because of the following positive effects? Select one: a. Reduced granulation tissue formation. b. Increased concentration of growth factors in the wound fluid. c. Enhance perfusion and migration of cells critical for healing. d. Prolonged cutaneous vasoconstriction.

c. Enhance perfusion and migration of cells critical for healing. Electrical stimulation is believed to have galvanotaxic effects (positive and negative charged cells are attracted toward opposite polarity in an electric field), stimulatory effect on fibroblasts, enhances blood flow and local tissue oxygen, & bacteriostatic & bactericidal effects.

You have a patient with a macular-papular redden rash to their lower leg. Which of the following dermatologic terms describes this assessment? a. Elevated, firm, palpable area deeper into the dermis >1 cm b. Flat, non-palpable area of discoloration >1 cm c. Flat and palpable skin discolorations <1 cm d. Elevated, firm, rough lesion with flat top

c. Flat and palpable skin discolorations <1 cm A macule is a flat, non-palpable circumscribed skin discoloration < 1 cm while a papule is a elevated, firm palpable area of skin <1 cm. A nodule is an elevated, firm palpable area deeper into the dermis >1 cm. A plaque is an elevated, firm rough lesion with flat top and a patch is a macule that is > 1cm.

The wound care nurse is following the TIME concept to promote wound healing. Which statement accurately describes this concept based on the tissue management step of the framework? Select one: a. A gauze dressing dampened with 1/4th strength Dakin's (hypochlorite) solution. b. Biofilm are beneficial to wound healing as they degrade the ECM. c. Hypergranulation tissue slows wound healing by preventing epithelialization. d. Hypertrophic scar tissue will resolve on its own when the incision heals.

c. Hypergranulation tissue slows wound healing by preventing epithelialization. Hypergranulation tissue is one type of inferior tissue that impedes wound healing by preventing re-epithelialization. The rest of the statements are all grossly false.

Which of the following statements is TRUE of the orthotopic neobladder? Select one: a. It is located in the abdomen. b. It is most often created from jejunum. c. It uses the native urethra as the outlet passageway. d. It is designed to hold a maximum of 150-250 cc urine.

c. It uses the native urethra as the outlet passageway.

Which of the following signs or symptoms are associated with diverticulitis? Select one: a. Fever, malaise, RUQ pain. b. Nausea/vomiting, RLQ pain. c. LLQ pain, fever, nausea/vomiting. d. Dysuria, bowel obstruction.

c. LLQ pain, fever, nausea/vomiting. Diverticulitis can cause severe abdominal pain, fever, nausea and a marked change in bowel habits. Pain or palpable mass are located in the LLQ.

What is the primary risk factor associated with Periwound MASD? a. Obesity b. Pouching system leakage c. Large amount of wound exudate d. Urinary incontinence

c. Large amount of wound exudate

You have a 4-year-old child that needs his low-profile gastrostomy tube replaced. What step would be MOST important when performing this procedure? Select one: a. Have parents give pain medication prior to procedure. b. Cleanse the skin with soap & water. c. Measure the length of the stoma tract with a measuring device. d. Use sterile gloves.

c. Measure the length of the stoma tract with a measuring device. Manufactures will provide a stoma-measuring device to be sure the correct size tube is used as the child grows and gains weight. If the small profile tube is too small it could cause pressure necrosis, pain, tenderness.

Including the patient and family in the plan of care improves patient adherence to the care plan because patients/families included in developing the plan of care are: a. Empowered to get out of the hospital sooner b. More knowledgeable about their condition and realize that some parts of the care plan are "optional" c. More apt to be adherent and have increased satisfaction d. Recognized as being above average intelligence and can choose to be non-compliant

c. More apt to be adherent and have increased satisfaction Including the patient and family in the plan of care improves patient engagement resulting increased adherence and satisfaction with care.

When preparing a 35-year-old male for a TPC (total proctocolectomy), he asks about sexual function after surgery. You explain that this procedure involves: Select one: a. Removal of the rectum only. b. Creation of a sigmoid colostomy. c. Narrow resection of the rectum. d. High incidence of erectile dysfunction.

c. Narrow resection of the rectum. Recall that a TPC (total proctocolectomy) is a narrow resection performed for benign disease that avoids nerve damage so that the pudendal nerve remains intact and penile erections are still possible.

While you are educating the staff at the NICU, one of the staff members wanted your recommendations on treating a pre-mature infant's skin stripping injury. Which of the following interventions would be the BEST choice to manage the skin stripping? Select one: a. Non-adherent dressing secured with paper tape. b. 2X2 gauze with antibiotic ointment. c. Non-adherent thin foam held in place with stretch netting. d. Application of stomahesive powder and alcohol free skin sealant.

c. Non-adherent thin foam held in place with stretch netting. The best choice is the use of a non-adherent thin foam which will absorb the weeping skin and secure with flexible netting

Mrs. Rose has an Indiana Pouch created for bladder cancer. She comes to your outpatient clinic and complains of increase mucus production with some increase odor, difficulty with catheterizations that are painful along with back pain. What should you do? Select one: a. Tell her to drink more fluids, change positions, and rotate catheter while inserting. b. Tell her to increase the size of the catheter and irrigate catheter with 30 ml of normal saline. c. Notify her surgeon and obtain a urine sample and send off for analysis, culture & sensitivity. d. Increase time between catheterizations and rinse catheter with hot water prior to use.

c. Notify her surgeon and obtain a urine sample and send off for analysis, culture & sensitivity. Mrs. Rose is having classic symptoms of pouchitis: so notify surgeon, obtain a urine analysis along with culture and sensitivity. Most likely the surgeon will order antibiotics.

Which of the following interventions would prevent a gastrostomy tube from clogging? Select one: a. Irrigate the tube daily with 10 ml of saline. b. Irrigate before medication administration. c. Only give liquid form of medications. d. Crush pills and dissolve in 10mL of Diet Pepsi.

c. Only give liquid form of medications.

An incarcerated bowel is an emergent complication associated with: Select one: a. Mucocutaneous separation. b. Stomal stenosis. c. Parastomal hernia. d. Peristomal pyoderma granulosum.

c. Parastomal hernia. A peristomal hernia may contain a loop of bowel in the herniation. If the mesentery of this segment of bowel is compromised, ischemia to the bowel segment can develop. When the patient has a peristomal hernia, it is important to teach them the s/s of ischemic bowel (abd. pain, n/v and/or change in stoma color)

ALL guidelines in WOC patient related areas, state that the following is critical to care: a. Patient/family development of strategies for home care b. Patient and family planning for early discharge c. Patient and family education d. Patient and family education classes on nutrition

c. Patient and family education ALL guidelines state that patient and family education is critical to care.

Which of the following reflect a long-term goal in a patient with a temporary ostomy? a. Patient can empty pouch independently b. Patient assists in pouch removal and application c. Patient is independent in ostomy care d. Patient performs a return demonstration on pouch change

c. Patient is independent in ostomy care Long-term goals would focus on independent ostomy care and adaptation. Short-term goals focus on the most pressing issue which in this case is to learn how to empty and change the ostomy pouch.

Research shows patient-centered communication can increase: a. The possibility of postoperative complications b. The patient's opportunity to participate in support groups c. Patient safety, outcomes and care experiences d. The patient's ability to provide self-care

c. Patient safety, outcomes and care experiences Research shows patient-centered communication can improve patient safety, outcomes and care experiences.

When the ostomy patient is taking corticosteroids and immunosuppressive agents, they are at risk for developing which of the following peristomal skin complications? Select one: a. Pyoderma gangrenosum. b. Pseudoverrucous lesions. c. Peristomal candidiasis. d. Caput medusae.

c. Peristomal candidiasis. Corticosteroids and immunosuppressants increase the risk of candidiasis in all patients. However, in the ostomy patient, it is usually peristomal candidiasis because they suppress the immune system.

Which of the following cells plays a central role in initiating the wound repair process? Select one: a. Endothelial cells. b. Fibroblasts. c. Platelets. d. Langerhan's cells.

c. Platelets. The clotting process (hemostasis phase), includes the rupturing of platelets, that triggers the entire wound healing cascade of events. When the platelets rupture, growth factors and cytokines are released.

Instructional approaches for the nursing team on how to perform WOC patient care include: a. YouTube presentations on WOC procedures b. Documentation of reading an article about WOC patient procedures c. Presentations of theory plus skills competency check-off d. Round table talks about safety ideas

c. Presentations of theory plus skills competency check-off Theory plus skill competency testing provides the best evidence of learning.

Your 80 y/o patient with diabetes is scheduled to have surgery to repair a hip fracture. What is the BEST POST-OP intervention to minimize her Surgical Site Infection (SSI) risk. Select one: a. Give supplemental oxygen prior to surgery. b. Maximize protein intake. c. Prevent hyperglycemia. d. Keep the patient warm.

c. Prevent hyperglycemia. Preventing postoperative hyperglycemia is possible the single most advantageous action to reduce SSI

What is the purpose for the prevention of MASD (Moisture Associated Skin Damage)? a. Prevent pressure forces b. Prevent shearing forces c. Promote skin integrity d. Promote skin maceration

c. Promote skin integrity

Which of the following statements is MOST appropriate in the application or removal of an adhesive type product? a. Apply adhesive product with tension or stretch b. Apply alcohol wipe to reduce oil on the skin prior to application c. Push and pull method with removal d. Quickly pull product off to reduce pain experience

c. Push and pull method with removal Correct application of adhesive product includes application of adhesive product without tension or stretch, apply firm and gentle tension to activate viscoelastic polymers to fit contours of epidermis. Correct removal of adhesive product includes using the push/pull method, low and slow approach with horizontal removal, stretch and release method for transparent films and adhesive removers for aggressive adhesive or residue.

The preferred abdominal location for an ileostomy in the adult is the: Select one: a. LLQ. b. LUQ. c. RLQ. d. RUQ.

c. RLQ. The preferred site for an ileostomy is the RLQ since that is the anatomic location of the distal ileum. The sigmoid colostomy is sited in the LLQ.

During a preoperative consultation for EMERGENCY ostomy surgery, the two PRIMARY objectives for the WOC Nurse are stoma site selection and: Select one: a. Explanation of how to empty the pouch. b. Demonstration of ostomy equipment. c. Reassurance and establishing rapport with the patient and family. d. Identification of discharge plans.

c. Reassurance and establishing rapport with the patient and family. Two primary objectives when conducting a pre-op visit for an emergency procedure is to (1) establish rapport and (2) mark the stoma site.

You are performing a wound assessment on a patient with a punch out appearing, deep ulcer, with small amount of serosanguinous drainage on the plantar aspect of their foot. You note that the depth is 0.5 cm, wound base is pale pink and probes to bone. The patient denies pain with palpation but you note mild periwound erythema. What would be your BEST response in developing the plan of care for this patient? Select one: a. Apply an antimicrobial hydrofiber dressing for 2 weeks. b. Apply topical antibiotic ointment with a gauze strip for 2 weeks. c. Refer to orthopedics for possible osteomyelitis as soon as possible. d. Apply cadexemer iodine for 2 weeks.

c. Refer to orthopedics for possible osteomyelitis as soon as possible. As described in the LEND Guidelines: '"Refer the patient for further evaluation if infection is suspected, such as when there is a positive probe to the bone."

Which of the following is an example of a patient center goal? a. Use of compression stockings in a patient with severe hand arthritis who lives alone b. Setting weekly wound center visits in a patient with limited finances who lives 70 miles from the center c. Reviewing written education materials with patient and family and asking for questions or clarifications d. Telling the patient, they must stop smoking or the wound will not heal

c. Reviewing written education materials with patient and family and asking for questions or clarifications Patient centered goals are determined together with the WOC nurse providing guidance, education, regarding the prevention or treatment plan. The patient must have the physical/mental/financial/social resources to follow the plan of care as well as desire.

Which of the following statements about a keloid is TRUE? Select one: a. It develops as a result of excess moisture. b. It remains within the boundary of the original injury. c. The scar becomes firm and fibrous & extends beyond the original wound margin. d. It will cause contractures.

c. The scar becomes firm and fibrous & extends beyond the original wound margin. Keloids are excessive scarring that appears firm and fibrous and extends beyond the borders of the original injury.

When selecting an abdominal stoma site, you should consider which of the following? Select one: a. The stoma should be sited lateral to the rectus muscle. b. The stoma should be located above the beltline. c. The stoma should be sited on the apex of the infraumbilical bulge. d. The stoma should be sited near the umbilicus.

c. The stoma should be sited on the apex of the infraumbilical bulge. Principles of stoma site selection include (1) through rectus muscle, (2) below beltline if at all possible, (3) away from umbilicus, creases or scars, and (4) on the apex of the infraumbilicul bulge.

A swab culture of the surface of the wound can misdiagnose a wound infection because: Select one: a. All wounds are infected. b. The surface organisms obtained in a swab of a desiccated surface are typically fungal. c. The surface organisms may not represent the organisms invading the wound tissue. d. The presence of leukocytes in the wound exudate neutralizes the invading organisms.

c. The surface organisms may not represent the organisms invading the wound tissue. While the swab culture is the most common method in obtaining a wound culture, it is riddled with potential error. All wounds are contaminated (not infected) and these surface organisms are not necessarily the organisms causing the infection. A wound biopsy would obtain a more accurate diagnosis, but is not practical in most settings.

Perineal skin care products are essential for the patient with incontinence because: a. Containment pads frequently case skin chaffing. b. They contain antifungal properties. c. They protect the skin from prolonged contact with moisture. d. They stimulate the rate of epidermal reproduction to strengthen the integrity of the skin barrier.

c. They protect the skin from prolonged contact with moisture.

Which type of wound exudate characteristic is most likely to cause periwound MASD? a. Serous b. Serosanguinous c. Thick purulent d. Sanguinous

c. Thick purulent Cloudy, viscous wound exudate is more likely to cause periwound MASD than clear, serous exudate. Chronic wound exudate has higher levels of proteolytic enzymes called MMPs (matrix-metalloproteases), purulent exudate has more bacterial toxins and inflammatory cytokines that digest or inflame periwound skin if the drainage sits on the skin for a prolonged period of time.

Pneumatosis intestinalis is a radiologic finding associated with: Select one: a. Hirschsprung's disease. b. Gut Malrotation. c. Diverticulitis. d. Necrotizing enterocolitis (NEC).

d. Necrotizing enterocolitis (NEC). Pneumatosis intestinalis means air in the bowel wall and is an indicator for pending bowel perforation associated with Necrotizing enterocolitis (NEC). A question on this topic is on almost every certification exam.

The procedure that is indicated for advanced and recurrent cancer of the cervix, vagina or vulva, and rectum which results in a sigmoid colostomy and ileal conduit is known as a/an: Select one: a. Lateral pelvic exenteration. b. Anterior pelvic exenteration. c. Total pelvic exenteration. d. Posterior pelvic exenteration.

c. Total pelvic exenteration. Removal of the bladder and rectum is required with advanced or recurrent cancer of the cervix, vagina or vulva and is called a total pelvic exenteration.

Which of the following patients is at risk for fistula formation after a surgical procedure? A patient who has: Select one: a. Experienced intraoperative hypothermia. b. Had preoperative hyponatremia. c. Undergone radiation therapy in the surgical area 20 years ago. d. A preoperative serum prealbumin level of 18.0.

c. Undergone radiation therapy in the surgical area 20 years ago. The combination of blood flow damaged by radiation and the interruption of blood flow by surgical procedures places the patient at a high risk for breakdown of anastomoses and consequently fistula formation.

What nutritional assessment data is MOST reflective for nutritional compromise? a. Albumin of 3.0 mg/dl b. Pre-albumin of 10 c. Unintentional weight loss >5% in last month d. Hgb of 9.5 mg/dl

c. Unintentional weight loss >5% in last month Unintended weight loss is a KEY indicator of malnutrition which should trigger a referral to a dietician for a comprehensive nutritional evaluation! Lab values indicate issues but can be altered by hydration, infection, acute or chronic disease. Lab values are not a 'Stand-alone" assessment for an individual's nutritional status.

Which of the following urinary diversions (continent or incontinent) will usually require a diaper or pouching to contain the urine? Select one: a. Indiana Pouch. b. Orthotopic Neobladder. c. Vesicostomy. d. Mitrofanoff procedure.

c. Vesicostomy. it is the only urinary diversion mentioned here that will require a diaper or pouching; the other diversions listed are all continent diversions.

You have a preterm neonate patient with a large surgical abdominal wound draining a large amount of exudate. What will you need to consider in wound management for this age patient? Select one: a. Eliminate the use of adhesives, and use a skin sealant (Eg. Skin Prep). b. Reduce the use of adhesives and use solvent to loosen tape. c. Weigh dressings and eliminate the use of adhesives. d. Skin sealants to peri-wound skin and the use of strong adhesives.

c. Weigh dressings and eliminate the use of adhesives. Your primary concern is to protect the pre-term neonate from fluid & electrolyte imbalance (weigh the dressings) because TEWL is very high with a higher surface to body weight ratio leading to dehydration (in addition to large wound output);

Medical adhesive products can produce allergic or contact dermatitis. Which of the following statements BEST demonstrates a skin reaction from the medical adhesive: a. Repeated exposure to medical adhesive products may produce erythema and edema b. Pustules with a hair piercing the center after shaving c. Well-defined vesicles (small blisters) found upon removal of adhesive product d. Wheal (raised edema of skin) anywhere on the body

c. Well-defined vesicles (small blisters) found upon removal of adhesive product Well-defined vesicles found upon removal of an adhesive product best demonstrate a skin reaction from a medical adhesive product. Folliculitis is described as inflammation of hair follicles which can represent an infection when associated with pustules. Rest of the statements are vague in describing an irritant or allergic skin reactions under an adhesive product.

Which of the following statements is TRUE? Venous dermatitis: Select one: a. indicates a wound infection. b. is characterized by hemosiderosis. c. results in erythema, crusting, scaling skin of the leg. d. is best managed with topical antimicrobial creams

c. results in erythema, crusting, scaling skin of the leg. Venous dermatitis results in erythema, crusting, scaling of the skin of the lower extremity.

which cells located in the dermis produce collagen (dermal building blocks) and elastin (tensile strength) a) leukotrienes b) mast cells c) macrophages d) fibroblasts

d) fibroblasts their main function is to make connective tissue and the extracellular matrix, and specifically produce collagen, elastin, and fibronectin

Which of the statements about ostomy equipment is correct? Select one: a. Skin barrier and pouch should be applied before the skin sealant (also known as a liquid skin barrier) dries. b. A skin barrier paste is essential to use for urinary and fecal ostomy care. c. A skin barrier powder should be applied liberally to the peristomal skin to prevent moisture buildup. d. A flexible skin barrier surface is usually indicated when the abdomen is firm.

d. A flexible skin barrier surface is usually indicated when the abdomen is firm. Match the abdomen to the flexibility of the pouching system. A firm abdomen requires a flexible pouching system; a soft abdomen usually requires a firm pouching system.

An essential feature of a urinary pouch during the IMMEDIATE postoperative period is: Select one: a. An opaque pouch film. b. A pouching adhesive surface that provides convexity. c. A one-piece pouch with an attached skin barrier. d. A sizeable (cut-to-fit) stoma opening.

d. A sizeable (cut-to-fit) stoma opening. The immediate postop pouch for a urinary AND fecal ostomy should be transparent and sizable

The absence of urine flow from a newly created urostomy is considered: Select one: a. Normal. b. A temporary issue that will resolve with additional intravenous fluids. c. Related to the stents. d. Abnormal... a medical emergency.

d. Abnormal... a medical emergency The flow of urine will be immediate and intermittent throughout a 24-hour day. There is no reservoir to store urine at this point since the bladder has been removed and a continent diversion has not been created

Which of the following statements is FALSE regarding MARSI (Medical Adhesive-related Skin Injuries): a. Common, usually painful, and almost preventable b. Types include contact dermatitis, maceration, skin stripping, tension blisters and folliculitis c. Securement of critical devices i.e. endotracheal tubes should be done with high adhesion products d. Adhesive products should be removed quickly at a 90-degree angle to generate a high peel force

d. Adhesive products should be removed quickly at a 90-degree angle to generate a high peel force Adhesive products should be removed at a low or horizontal angle, and the skin adjacent to the peel line should be supported to prevent pulling. Peel force is associated with skin injury. Removing at a 90-degree angle (perpendicular) to the skin surface generates a high peel force thereby increasing the risk for skin stripping or tear. Rest of the statements are correct regarding MARSI.

Patients undergoing myocutaneous flap closure of a pressure ulcer are best managed in the initial postoperative period on what type of support surface? Select one: a. An alternating air mattress with frequent position changes. b. Any surface as long as they are turned every 2 hours. c. A water-flotation device. d. An air-fluidized bed with non-shear surface.

d. An air-fluidized bed with non-shear surface. The standard of care for post flap care is to place the patient with paraplegia or quadriplegia on an air-fluidized bed with a non-shear surface such as the "Clinitron".

Mr. Buss has undergone preoperative radiation therapy for rectal cancer. He has developed profuse diarrhea, rectal pain, fecal urgency with incontinence, and nausea. Which of the following is not part of your plan of care in managing these symptoms? Select one: a. Nutritional support. b. Anti-diarrheals. c. Anti-inflammatory agents. d. Anti-depressants.

d. Anti-depressants.

The nurse assessing the pain of a patient with chronic leg ulcers documents a moderately severe pain level of seven on a scale of one to ten. In the following list, which intervention would NOT be an appropriate adjunct to his scheduled pain medication? Select one: a. An option to call a "time out" during wound relate procedures. b. Diversional activities i.e. slow breathing, music. c. Implementation of a non-adhesive dressing. d. Application of silver sulfadiazine to reduce bioburden.

d. Application of silver sulfadiazine to reduce bioburden. Effective management of wound related pain involves appropriate use of analgesic and adjuvant medications, as well as adjunctive non-pharmacologic therapies, including the option to call "time out" during wound care and diversional activities. Silver sulfadiazine is not appropriate because there was no mention in the case study that the wound was critically colonized.

To reduce the potential for wound dehiscence in the bariatric client following abdominal surgery, the nurse should: Select one: a. Use Montgomery straps to secure the ABD pad over the surgical incision. b. Move the patient as little as possible for the first 48 hours. c. Discourage pulmonary exercise such as cough and deep breathing. d. Apply a surgical binder to support the surgical area.

d. Apply a surgical binder to support the surgical area. Adipose tissue is not well vascularized thus applying surgical binder will provide support and reduce tension upon the suture lines

Mrs. Jones has an ulcer located on the dorsal surface of the third toe on the left foot. The ulcer is 1 cm in diameter, with a dry, pale pink wound bed. She reports significant pain in the ulcer and denies any history of diabetes. What is the MOST likely cause of this ulcer? Select one: a. Venous hypertension. b. Neuropathy. c. Pressure. d. Arterial insufficiency.

d. Arterial insufficiency. Classic symptoms for arterial insufficiency...dry, pale wound bed, very painful, tips of digits.

A complication of diverticular disease that almost always requires surgical intervention. Select one: a. Intestinal inflammation. b. Micro-perforation. c. Diverticulum formation. d. Bowel perforation.

d. Bowel perforation.

You are assessing a skin lesion to the upper arm that is elevated, serous fluid filled beneath the epidermis that is 2X3 cm. The correct dermatologic term for this is which of the following: a. Vesicle b. Pustule c. Cyst d. Bulla

d. Bulla The correct term for a clear fluid filled blister > 1cm is a bulla. Vesicles are blisters < 1cm while a pustule is a vesicle filled with purulent fluid. A cyst is an encapsulated lesion in the dermis or subcutaneous tissue filled with semi-solid or liquid material.

An Ileal pouch anal anastomosis (IPAA) which is also referred to as the ileoanal reservoir is indicated for which of the following disease processes? Select one: a. Irritable bowel syndrome and Crohn's Disease. b. Colorectal cancer and chronic ulcerative colitis. c. Crohn's Disease and chronic ulcerative colitis. d. Chronic ulcerative colitis and familial adenomatous polyposis.

d. Chronic ulcerative colitis and familial adenomatous polyposis. IPAA is the construction of an ileal pouch within the pelvis. This would not be done for malignancy or a fistulizing disease (Crohn's Disease) where a hole could occur in the pouch.

Which of the following statements about Crohn's disease and/or chronic ulcerative colitis is true? Select one: a. Both Crohn's Disease and chronic ulcerative colitis are characterized by patchy mucosal involvement and skip lesions. b. Crohn's Disease is limited to the colon and rectum. c. Chronic ulcerative colitis is characterized by transmural involvement of the bowel wall. d. Chronic ulcerative colitis begins in the rectal area and progresses backward toward the ileocecal valve.

d. Chronic ulcerative colitis begins in the rectal area and progresses backward toward the ileocecal valve. Crohn's disease is a transmural disease that has a patchy distribution through the GI tract thus creating skip lesions; it commonly occurs initially in the ileal cecal area and can be mis-interpreted initially as appendicitis. Chronic Ulcerative Colitis always begins in the rectum then progresses back toward the ileocecal valve in a continuous fashion so there are no "skip" lesions (no healthy mucosa between diseased mucosa since it is all diseased).

Structures of the dermis include: a. Collagen, melanocytes, keratinocytes, & sweat (sebaceous) glands b. Fat, vasculature, hair follicles, & melanocytes c. Connective tissue, fat, keratinocytes, & elastin proteins d. Collagen, elastin proteins, hair follicles, sweat (sebaceous) glands

d. Collagen, elastin proteins, hair follicles, sweat (sebaceous) glands The dermis contains collagen, elastin proteins, nerves and vasculature as well as connective tissue, hair follicles, sweat and sebaceous glands. Melanocytes and keratinocytes are in the epidermis. Fat is in the hypodermis or subcutaneous tissue.

Patients with acute radiation-induced colitis would likely not demonstrate the following symptom: Select one: a. Abdominal cramping. b. Diarrhea. c. Nausea/vomiting. d. Constipation.

d. Constipation.

The Health Belief Model is designed to: a. Determine what the patient believes about the likelihood that he will recover from a health care problem b. Identify what the patient believes about potential sexual dysfunction problems he might encounter after surgery c. Identify problems the patient thinks will prevent adherence to a protocol of care following hospitalization d. Determine the likelihood that the patient would engage in health-promoting behaviors

d. Determine the likelihood that the patient would engage in health-promoting behaviors The Health Belief Model is designed to determine the likelihood that the patient will engage in health-promoting behaviors. One of the most widely used models of health care behavior used. It was originally formulated to model the adoption of preventive health behaviors in the United States, the HBM has been successfully adapted to fit diverse cultural and topical contexts.

A 28-year-old female has a permanent ileostomy and you notice ulceration at 9 o'clock in the peristomal area 7 mm from the base of the stoma. The ulcer is producing fecal material. Without knowing her history, what is this describing? Select one: a. Candidiasis due to steroids. b. Familial adenomatous polyposis. c. Peristomal pyoderma gangrenosum. d. Enterocutaneous fistula.

d. Enterocutaneous fistula.

Which cells located in the dermis produce collagen (dermal building blocks) and elastin (tensile strength)? Select one: a. Leukotrienes. b. Mast cells. c. Macrophages. d. Fibroblasts.

d. Fibroblasts. Fibroblasts' main function is to make connective tissue and the extracellular matrix, specifically the production of collagen (dermal building blocks) and elastin (tensile strength) and fibronectin, which binds extracellular matrix proteins.

Which of the following foods is known to cause considerable odor for the patient with a urinary diversion? Select one: a. Beets. b. Cheese. c. Greasy, French fries. d. Fish.

d. Fish. fish, asparagus, vitamins are notorious for urinary odor.

The patient with chronic ulcerative colitis will commonly exhibit all of the following EXCEPT: Select one: a. Diarrhea. b. Anemia. c. Bloody stools. d. Fistula formation.

d. Fistula formation. Fistula formation is typical with Crohn's disease (never Chronic Ulcerative Colitis).

Which area of the body is the MOST difficult to offload using a pressure redistribution surface? Select one: a. Sacrum b. Occiput c. Trochanter d. Heel

d. Heel The heel is the most difficult area of the body to protect because of the greater degree of mechanical loading and vessel compression over an area covered with only a thin layer of overlying soft tissue and the small curvature of the bony prominence.

A patient has an acute full-thickness wound due to trauma. What order of healing would occur with this wound? Select one: a. Inflammatory, hemostatis, granulation tissue formation, re-epithelialization. b. Re-epithelialization, granulation tissue formation, hemostasis, remodeling. c. Hemostatis, resurfacing, scar formation, remodeling. d. Hemostatis, inflammation, proliferation or regeneration, maturation or remodeling phases.

d. Hemostatis, inflammation, proliferation or regeneration, maturation or remodeling phases. The deeper the tissue layers affected, the greater number of wound healing mechanisms required. Full thickness wounds will experience all four phases of wound healing: hemostasis, inflammation, proliferation where granulation tissue is formed, re-epithelialization, and finally the maturational or remodeling phase.

Of the following dressings, which would be most appropriate for a shallow, clean, granulating wound that measures 2 cm X 1.5 cm and has minimal exudate? Select one: a. Calcium alginate. b. Wet-to-dry gauze. c. Enzymatic debrider. d. Hydrocolloid dressing.

d. Hydrocolloid dressing. The goal for this wound is to maintain moisture and cover the shallow ulceration. Hydrocolloid is best for shallow wounds with no to low volume of exudate.

Autolysis in a non-exudative pressure ulcer with an eschar covered wound base is most efficiently achieved with which of the following techniques? Select one: a. Whirlpool and frequent irrigation. b. Wet to dry dressing changes twice daily. c. Alginate dressing covered with a gauze. d. Hydrogel covered with a transparent film.

d. Hydrogel covered with a transparent film. The goal for this ulcer is to donate moisture for autolytic debridement to occur. Moisture is applied reliably and consistently with transparent dressings and hydrogel. Since this is a very dry wound bed, it is most appropriate to use a hydrogel dressing.

Six months postoperatively, you can expect the patient with which of the following urinary diversions to be using bedside drainage at night? Select one: a. Indiana pouch b. Mitrofanoff c. Koch pouch d. Ileal conduit

d. Ileal conduit Only the ileal conduit requires a night time drainage container because the other diversions are continent diversions

One of the important features of an endoscopy gastrostomy procedure, compared to a surgically placed gastrostomy, is that the endoscopy gastrostomy procedure allows feeding to occur: Select one: a. Within 8-12 hours. b. Within 24 hours. c. After 48 hours. d. Immediately.

d. Immediately.

The skin around the stoma should be: Select one: a. Denuded because of the shaving and surgical scrub. b. Pale as compared with the rest of the abdomen. c. Erythematous because of the pouch adhesive. d. Intact and healthy.

d. Intact and healthy.

A two-year-old female toddler presents in the emergency department with abdominal pain, vomiting, palpable abdominal mass in RUQ, and bloody stool that looks like red currant jelly. What condition may be present? Select one: a. Volvulus b. Colonic inertia c. Diverticulitis d. Intussusception

d. Intussusception

The wound care nurse is monitoring the cleansing process of a necrotic wound. What technique is recommended? Select one: a. Use cytotoxic solution to gently flush the wound. b. Irrigate the wound with normal saline at <5 psi force. c. Vigorously flush the wound with diluted hydrogen peroxide. d. Irrigate the wound with 4 to 15 psi force.

d. Irrigate the wound with 4 to 15 psi force. Cleansing of necrotic tissue requires irrigation rather than gentle flushing because irrigation is much more effective in reducing bacterial loads and loosening avascular tissue/debris. Recommended force is 4-15 psi (pounds per square inch). Hydrogen peroxide and cytotoxic solutions should not be used.

What is the purpose of the support bridge that may be placed under the loop stoma? Select one: a. It assists the surgeon to know where both ends of the bowel are located in anticipation for stoma takedown. b. It takes pouching the stoma easier. c. It provides an easy access to the stoma for irrigation. d. It supports the bowel until the stoma heals to the abdominal skin.

d. It supports the bowel until the stoma heals to the abdominal skin. Loop stomas are usually temporary and are therefore, not constructed like other stomas. The rod supports the stoma during healing. It is usually removed within 5 - 7 days.

A heel ulcer with a non-infected stable eschar is BEST managed by which of the following: Select one: a. Hydrogel application. b. Enzymatic debridement agent. c. Sharp instrumental debridement. d. Leave open to air.

d. Leave open to air. Stable eschar without drainage should be left intact as defined in webcast Wound Bed Preparation. These wounds are often left open to the air to keep the eschar dry so they will shrink, demarcate, and slowly lift off independently when the underlying tissue has healed. The dry eschar serves as a protective cap to outside offending pathogens. If redness or drainage begins, then this wound no longer has a stable eschar and therefore warrants further evaluation and treatment.

John is 15-year-old with a new loop ileostomy with an IPAA for refractory chronic ulcerative colitis. This is your first post-operative teaching session. What assumptions can you make about John's primary concerns regarding is new ostomy based upon Erikson's psychosocial phases? Select one: a. Poor self-esteem and less autonomy b. Fear of loss of occupation and role changes c. Limited activities and inhibition d. Limited peer relationships and poor body image

d. Limited peer relationships and poor body image Since John is a teenager, his primary concerns for his age are identity and role confusion that may lead to limited peer relationships and changes in his body image. He may be angry and uncooperative in learning self-care initially.

When the skin around a wound develops a whitish appearance, it can be described as: a. Desiccated b. Excoriated c. Indurated d. Macerated

d. Macerated Maceration has the appearance of whitish tissue that is also wrinkled or prune-like; it occurs when moisture sits on the skin for extended periods of time.

47% of all pressure ulcers occur over the ischial tuberosities and sacrum. What PRIMARY prevention intervention would the nurse institute to address ischial tuberosities based on this statistic? Select one: a. Have the patient lie supine in bed to sleep. b. Pad the bony prominences of the heels with foam. c. Tell the patient to stay out of the wheelchair as much as possible. d. Maintain the wheelchair and seat cushion in proper working order.

d. Maintain the wheelchair and seat cushion in proper working order. Prescribed equipment must be maintained in order to perform as intended

You have applied a skin sealant under the adhesive surface of a fistula pouch. You indicate to the nursing staff that the rationale for this intervention is to protect the skin from: Select one: a. Chemical injury. b. Allergy to adhesives. c. Contact with skin barrier pastes. d. Mechanical injury.

d. Mechanical injury The skin sealant or liquid skin barrier is a thin liquid copolymer applied to the skin to protect it from skin stripping (a mechanical injury).

The procedure in which the bladder can remains intact and the appendix is used as a channel for the urine to exit through the abdomen is known as: Select one: a. Indiana pouch. b. Orthotopic neobladder. c. Vesicostomy. d. Mitrofanoff procedure.

d. Mitrofanoff procedure. The Mitrofanoff procedure may be used with the patient's own bladder (most often they have neurogenic bladder and are wheelchair bound but have use of their hands). By leaving the bladder in place and creating an abdominal opening to use to drain the urine, the patient can easily do intermittent selfcatheterization.

Mrs. Wells, 86 yrs. old, is being discharged from acute care to homecare services to aide in the recovery of a stroke. She has some urinary frequency, urge incontinence, right sided weakness, uses a walker without assist and lives alone. Which of the following represents a structural barrier? a. Use of body worn absorbent products b. Living home alone c. Age d. Need for safety bars in bathroom

d. Need for safety bars in bathroom Structural barriers in following the plan of care include patient safety issues such as environmental adaptations (need for ramps, wider doorways, running & clean water, BR), equipment needs (w/c, hospital bed, Hoyer lift, safety grab bars), care transitions from one setting to another (Hand Off communications) and past experiences.

Which of the following dermatologic configurations describe 'Coin like' skin lesion? a. Arcuate b. Linear c. Annular d. Nummular

d. Nummular Nummular describes a coin shape (raised round or oval redden) lesion configuration. Arcuate describes curved shape lesion; annular describes a ring with central clearing. Linear describes a line shape.

Which single risk factor is MOST likely to lead to SSI (surgical site infection)? Select one: a. Pain. b. Hypothermia. c. Tissue hypoxia. d. Obesity.

d. Obesity. Obesity is and independent predictor of SSI (tissue hypoxia, hyperglycemia, factors affecting vasoconstriction-pain, cold, volume loss).

Later in the postop recovery process it is important to provide instructions on pelvic muscle exercises to the patient with which of the following surgical procedures? Select one: a. Indiana pouch. b. Mitrofanoff procedure. c. Vesicostomy. d. Orthotopic neobladder.

d. Orthotopic neobladder. The Valsalva maneuver and relaxation of pelvic muscles to void will be important to perform since the neobladder will not have the ability to contract as the "real" bladder does. Pelvic muscle exercises will help the patient improve their ability to contract the external sphincter (bladder neck) to store urine and accurately relax the pelvic floor when voiding

Mr. Smith has had a sigmoid colostomy that he irrigates every day with regular results. Lately, he has noticed a bulging around the stoma and consistently poor returns of the irrigation fluid and feces. What complication is Mr. Smith likely experiencing? Select one: a. Stoma prolapse. b. Pseudoverucous lesions. c. Stoma necrosis. d. Peristomal hernia.

d. Peristomal hernia.

You have a patient with a Stage 3 pressure injury to R. ischial tuberosity that is covered with 100% necrotic tissue. Prior to performing conservative sharp debridement, what factor is the least of importance in the ability to perform conservative sharp debridement? Select one: a. Coverage under the nurse's state practice act. b. Formal didactic and clinical skills validated. c. Policy, procedure, & protocol in place. d. Presence of loose, avascular tissue in the wound.

d. Presence of loose, avascular tissue in the wound. Nurse Practice Act defines a nurse's scope of practice in each state so a WOC nurse must check that conservative sharp debridement is within their scope of practice. Most employers require and WOCN recommends additional class and clinical work to verify that a WOC is competent in conservative sharp debridement. Policy, procedures, & protocols should be in place prior to instituting conservative sharp debridement within your practice setting. The last option is of least importance because the first three must be met prior to performing the debridement.

Which of the following foods contain the highest levels of fiber? a. Canned vegetables, yogurt, cheese b. Fruit juice, fish, eggs c. Applesauce, pudding, white rice d. Prunes, figs, oat bran

d. Prunes, figs, oat bran Foods high in fiber include prunes, figs, oat bran, dried fruits, whole grain products, nuts, raw veggies and fruits, dried beans, peas, lentils, coconut and popcorn

Hyperbaric oxygen treatments (HBOT) are indicated for which of the following conditions? Select one: a. Pressure injury. b. Refractory chronic venous ulcer. c. Epidermolysis Bullosa (EB). d. Radiation induced necrotic wounds.

d. Radiation induced necrotic wounds Key wound related indicators for HBOT are radiation induced necrosis, ischemic wounds, diabetic wounds with deep tissue or bone infections and also for the compromised surgical flaps.

The nursing staff calls you because a patient suffered an extravasation of Vancomycin at a peripheral IV site, your immediate instructions are to: Select one: a. Call the surgeon to schedule an immediate surgical debridement and skin grafting. b. Apply pressure to the IV site for 30 minutes. c. Flush IV site with Normal Saline and apply warm compress to site. d. Recommend topical wound therapy once residual drug is aspirated.

d. Recommend topical wound therapy once residual drug is aspirated. Removal of residual drug by aspiration is recommended to reduce exposure of tissue to the vesicant, avoid pressure, and elevate the affected limb for 24 hours.

You are aware of the skin and wound healing age-related changes that occur. Which of the following is an age-related change: Select one: a. Increased bond between the dermal-epidermal junction. b. Thickening of the dermis and subcutaneous tissues. c. Increased ability to manage the inflammatory response. d. Reduced ability to manage the inflammatory response.

d. Reduced ability to manage the inflammatory response. All of the statements are false regarding age-related changes of the skin in the geriatric population except their ability to have an adequate inflammatory response is reduced (not enhanced) due to fewer number of macrophage, mast & T-cells.

You are called into a care conference to help the Skin Care Team determine a plan of care to promote bariatric skin integrity. What characteristic listed is NOT a risk factor for alteration in skin integrity for those who suffer obesity? Select one: a. Increased perspiration b. Increased TEWL (transepidermal water loss) leading to pH elevation c. Increased inflammation due to elevated gland activity d. Reduced bacterial load on his skin

d. Reduced bacterial load on his skin Bacterial load is actually increase in those who suffer obesity especially in the skin folds due to increase moisture, elevated pH reduces the acid mantle of the skin thus proliferation of pathogens is elevated, and pathogens grow more readily in a dark, moist environment in skin folds and intertriginous areas.

The new patient admitted to the LTAC (Long-term Acute Care) has a stage 3 pressure injury with 100% granulation tissue and moderate amount of exudate. The wound edge is well defined with a rolled edge from 12 to 5 o'clock and hyperkeratotic edge from 5 to 12 o'clock. The most appropriate intervention you select for this wound is the following: Select one: a. Calcium alginate. b. Silver hydrofiber. c. Hydrocolloid. d. Referral to open up the edges.

d. Referral to open up the edges. Wound edges that are rolled (epibole) is a sign of long present wound and will not go onto resolution (no matter what topical dressing is selected) until the edges are opened with sharp debridement due to contact inhibition of the epithelial cells.

MARSI can be prevented with the proper technique of adhesive removal by: a. Pulling the adhesive off quickly b. Removing the adhesive at a 90-degree angle c. Using alcohol to release the adhesive d. Removing the adhesive in the direction the hair grows

d. Removing the adhesive in the direction the hair grows MARSI can be prevented with the proper technique of adhesive removal by removing the adhesive in the direction the hair grows. Use 2 hands and remove the adhesive at an angle parallel to the skin avoiding a 90° angle. Remove adhesives slowly and horizontally. Consider use of an adhesive remover.

Mr. Neverland has a small, full thickness neuropathic foot ulcer that is not progressing for the past 2 weeks although arterial circulation is adequate, no signs of infection, offloading with total contact casting & topical wound management have been implemented. What is the most likely reason for his wound to be stagnant? Select one: a. Detached wound edge b. Wound colonization c. Compromised arterial circulation d. Senescent cells

d. Senescent cells If all 3 principles of chronic wound management is being addressed, the patient is offloaded and adequate circulation then senescent cells or malignancy is the MOST likely cause.

Which of the following dressings is most appropriate to use in a pressure ulcer with a granulation tissue filled wound bed measuring 3cm X 5 cm X 2.5 cm with undermining that extends 4 cm from 6 to 9 o'clock? The wound has become increasingly exudative in the past 3 weeks. Select one: a. Dakin's wet to dry gauze. b. Chemical debriding agent. c. Hydrogel impregnated gauze. d. Silver impregnated alginate rope.

d. Silver impregnated alginate rope. The wound has exudate so we need something to absorb the exudate and fill in the depth and undermining. In addition, the volume of exudate has been increasing which is an indicator of critical colonization. Therefore the wound nurse should choose to use an alginate and specifically a silver impregnated alginate rope to not only fill the dead space and absorb exudate but also control bacterial load.

Moist, denuded perineal skin resulting from exposure to urine and stool is BEST treated with the use of: a. Cornstarch and ointments b. Antifungal powders and liquid barrier films c. Moisturizer and perineal cleansers d. Skin barrier powder and ointments

d. Skin barrier powder and ointments When the skin is denuded and moist you must use a skin barrier powder to adhere and dry up the moisture, then apply a skin barrier ointment...or you can a skin barrier paste

When you read in the postoperative surgical report that the stoma was primarily matured, you can expect which of the following? Select one: a. The stoma will be opened at the bedside with electrocautery. b. The stoma has been present for at least 3 months. c. The stoma is ready for sutures at the mucocutaneous junction to be removed. d. The bowel was everted and sutured to the abdominal surface during surgery.

d. The bowel was everted and sutured to the abdominal surface during surgery. Primarily matured, Brooke style stoma, and everted are all terms to describe the technique where the bowel is cuffed like a sock (thus exposing the mucosal surface) and sutured to the skin at the end of the operation.

Which of the following statements about contact casting is correct? Select one: a. The contact cast is a multi-layer graduated compression wrap. b. The patient can be instructed to replace the contact cast every 7 days. c. It may be used when the ulcer is infected. d. The contact cast redistributes the weight of the diabetic foot.

d. The contact cast redistributes the weight of the diabetic foot. A contact cast is a cast used with neuropathic ulcers that redistributes the weight of the foot so that the pressure over a specific area on the plantar surface of the foot is relieved; it is applied by a nurse or physician or PT who is specifically trained in application technique and can generally be changed once per week; it is contraindicated if the wound has untreated infection.

MASD from intertriginous dermatitis (ITD) is differentiated from incontinence associate skin damage (IAD) in that: a. IAD only occurs in skin folds b. IAD may be complicated by bacterial or fungal infection c. ITD only occurs between the buttocks d. The source of moisture and location are different

d. The source of moisture and location are different

You have determined that Mr. Benn has classic symptoms of venous dermatitis: redness, scaling, crusting, & itching. What treatment is best to manage his symptoms in the following? Select one: a. Triple antibiotic ointment. b. Lanolin cream bid. c. Antifungal cream twice daily. d. Topical corticosteroid daily for 1 week.

d. Topical corticosteroid daily for 1 week. A mild potency of a topical corticosteroid may be necessary short term (1-2 weeks). Antibiotic ointments and creams with lanolin should not be used on patients with venous insufficiency due to sensitization of the skin to these products.

Preventive measures for patients with high risk for skin tears include: a. Shoes, protective sleeves & use of fingers to reposition b. Shoes, protective sleeves & avoidance of assistive devices c. Twice a day moisturizer, fluids, and side rails in the up position d. Twice a day moisturizer, fluids, and side rails in the down position

d. Twice a day moisturizer, fluids, and side rails in the down position Preventive measures for patients with high risk for skin tears includes methods of providing a safe environment, measures to maintain nutrition and hydration as well as protect form self-inflicted and caregiver injury.

Expert opinion list which of these interventions as effective in the management MASD caused by Intertriginous Dermatitis (ITD)? a. Use of talc or cornstarch powder in skin folds b. Place bed or bath linens (sheets, pillowcases, wash cloths) in skin folds c. Place gauze or paper towels in skin folds d. Use of breathable barrier skin protectant (i.e. dimethicone, skin sealant)

d. Use of breathable barrier skin protectant (i.e. dimethicone, skin sealant) Expert opinion currently recommends using a breathable barrier product that helps reduce the skin to skin friction as well as repels the moisture. Cornstarch and talc powders tend to gum up in the skin fold and promote bacterial/fungal growth; bath linens and other products such as gauze, paper towels that do not have a high level of moisture vapor permeability are not affective in wicking moisture from skin folds and may add friction.

Mr. Hanson has an enterocutaneous fistula that you plan to pouch. What 4 criteria will be essential when selecting a pouching system for him? Select one: a. Volume, consistency, need for access and pH of effluent. b. Need for access, volume, odor, pH of effluent. c. Size of fistula opening on the skin, odor, consistency of effluent and opaque pouch. d. Volume of effluent, size of fistula opening on the skin, odor and need for access.

d. Volume of effluent, size of fistula opening on the skin, odor and need for access. #1) Size of fistula opening determines size of surface area of pouch barrier, 2) Volume of exudate determines the size of the pouching system or need for straight drainage 3) Odor will drive you to contain the output by pouching even if the output is very low simply because the pouch will contain the odor. 4) Need for access - if you need to access the fistula while the pouch is intact, an access window or a 2-pc ostomy pouching system will be need

In the following statements, which is objective data found in the focus physical exam? a. Patient divulges smoking cannabis b. You request a fluid and bladder diary c. Chart review notes the patient uses a walker Incorrect d. Wound dimensions and characteristics

d. Wound dimensions and characteristics Objective data is gathered when the WOC nurse performs a focus physical exam (observation, percussion, palpation, measurements), as well as diagnostics, labs, & conduction of risk assessment tools. Anything the patient tells you is considered subjective data and is gathered during the patient interview.


Ensembles d'études connexes

Abeka 6th Grade, Scienct Test 4,(Ch. 1-2)

View Set

Q1 (10%) Prematurity/Special Needs

View Set

Chapter 8: Other Fire Prevention Functions

View Set

BUS 496 Strategic Management Ch. 5

View Set

Food Animal Exam #2 Practice Questions (Swine, Beef, & Small Ruminants)

View Set

Peds Ch. 46 Cerebral Dysfunction

View Set

Anatomy Chapter 13 - Innervation of specific body regions - Cervical plexus and the neck

View Set